Hidden knowledge

by Judith Curry

I stumbled across this essay by Michael Nielsen entitled “Science Beyond Individual Understanding,” which I think is very relevant to the climate problem.

Hidden knowledge is a concept derived from a paper by economist Frederick entitled “Use of Knowledge in Society.”

The peculiar character of the problem of a rational economic order is determined precisely by the fact that the knowledge of the circumstances of which we must make use never exists in concentrated or integrated form, but solely as the dispersed bits of incomplete and frequently contradictory knowledge which all the separate individuals possess.  Or to put it briefly, it is a problem of the utilization of knowledg not given to anyone in its totality.

Nielsen’s essay adapts the concept of hidden knowledge to science:

In economics, many basic facts, such as prices, have an origin which isn’t completely understood by any single person, no matter how bright or well informed, because none of those people have access to all the hidden knowledge that determines those prices.

By contrast, until quite recently the complete justification for even the most complex scientific facts could be understood by a single person.

Science is no longer so simple; many important scientific facts now have justifications that are beyond the comprehension of a single person.

JC:  This is certainly true of the climate problem in all its complexity.

No single person understands all of this, except in broad detail. 

Instead, there will be a large group of people who collectively claim to understand all the separate pieces that go into the discovery, and how those pieces fit together.

JC: This would explain the IPCC “consensus” and the large number of people that support it.

Two clarifications are in order. First, when I say that these are examples of scientific facts beyond individual understanding, I’m not saying a single person can’t understand the meaning of the facts.  I’m talking about a deeper type of understanding, the understanding that comes from understanding thejustification of the facts.

JC:  Yes, this aptly describes the missing arguments and justifications from the IPCC report, which instead substitutes citations of papers and lists of evidence, summarized by a confidence level derived from expert judgment.

Second, I don’t mean that to understand something you need to have mastered all the rote details.

JC:  The emphasis of a certain element of the climate blogosphere on  factoids and information and the dismissal of anyone who makes a misstatement is not useful in the context of understanding.

How do we know whether [scientific discoveries are] right or wrong? The traditional process of peer review and the criterion of reproducibility work well when experiments are cheap, and one scientist can explain to another what was done. But they don’t work so well as experiments get more expensive, when no one person fully understands how an experiment was done, and when experiments and their analyses involve reams of data or ideas.

JC: Because of the complexity of many climate studies, peer review is typically cursory in its ability to uncover methodological or logical problems.

Might we one day find ourselves in a situation like in a free market where systematic misunderstandings can infect our collective conclusions? How can we be sure the results of large-scale collaborations or computing projects are reliable? Are there results from this kind of science that are already widely believed, maybe even influencing public policy, but are, in fact, wrong?

JC:  need I actually say it?

But such collaborations will be no good if we can’t assess the reliability of the results. And it would disastrous if erroneous results were to have a major impact on public policy. We’re in for a turbulent and interesting period as scientists think through what’s needed to arrive at reliable scientific conclusions in the age of big collaborations.

JC:  this seems to be what Steve McIntyre is going after, in terms of the auditing concept and engineering-style explanation, and making explicit the “hidden knowledge”.

p.s.  more from Michael Nielsen this weekend.


368 responses to “Hidden knowledge

  1. How can we be sure the results of large-scale collaborations or computing projects are reliable? Are there results from this kind of science modeling that are already widely believed, maybe even influencing public financial policy, but are, in fact, wrong?

    AIG might be able to answer that.

    • “How can we be sure the results of large-scale collaborations or computing projects are reliable?”

      My experience is that the larger the budget for a project and the greater the number of collaborators**, the less reliable will be the conclusions.

      Shortly after neutron repulsion was discovered and reported at the 32nd Lunar Science Conference in March 2001 [1] as the solution to the “Solar Neutrino Puzzle”, a large army of scientists (~178, as I recall) funded by DOE published a paper claiming that most solar electron neutrinos just oscillate away before they reach our detectors!

      They did not mentioned that the “Solar Neutrino Puzzle” had already been solved, and to this day DOE ignores neutron repulsion [2] – the source of nuclear energy that powers the Sun, the cores of other stars and galaxies, exerts dominant control of Earth’s climate, and maintains our very lives.

      [1] http://xxx.lanl.gov/pdf/astro-ph/0411255
      [2] http://arxiv.org/pdf/1102.1499v1

      **Perhaps the basic problem is that large-scale collaborations – like the Large Hadron Collider (LHC) and the Sudbury Neutrino Observatory – are directed by bureaucrats in funding agencies, rather than by scientists who tend to “fly by the seats of their pants, letting each new finding determine the future direction of the project”.

      • Oliver,

        Considering that the collider experiments are on molecules that do not rotate like a planet or sun.
        Good intentions on bad science.

      • The scientists themselves are not to blame, Joe.

        The problem is higher in the food chain, in the offices of federal research agencies that are trying to direct science to discoveries that will generate more funds for their agency in next year’s budget.

        Eisenhower saw this problem coming in January of 2001:

        http://mcadams.posc.mu.edu/ike.htm

        “Akin to, and largely responsible for the sweeping changes in our industrial-military posture, has been the technological revolution during recent decades.”

        “In this revolution, research has become central, it also becomes more formalized, complex, and costly. A steadily increasing share is conducted for, by, or at the direction of, the Federal government.

        “Today, the solitary inventor, tinkering in his shop, has been overshadowed by task forces of scientists in laboratories and testing fields. In the same fashion, the free university, historically the fountainhead of free ideas and scientific discovery, has experienced a revolution in the conduct of research. Partly because of the huge costs involved, a government contract becomes virtually a substitute for intellectual curiosity. For every old blackboard there are now hundreds of new electronic computers.”

        “The prospect of domination of the nation’s scholars by Federal employment, project allocations, and the power of money is ever present – and is gravely to be regarded.

        “Yet, in holding scientific research and discovery in respect, as we should, we must also be alert to the equal and opposite danger that public policy could itself become the captive of a scientific-technological elite.

    • Thank you, Professor Curry, for this open discussion on “Science Beyond Individual Understanding”- a dangerous falsehood promoted by those who twisted government science on climate into a propaganda machine.

      The same mob mentality that produced the “Climategate” scandal is alive and well in almost all branches of government science: e.g., Astronomy, Astrophysics, Cosmology, Nuclear, Particle, Solar and Space Physics.

      Count, for example, the number of scientists and research institutions associated with this important report from Fermilab on their discovery of an unexpected and powerful, repulsive nuclear force in nature:

      http://arxiv.org/pdf/1104.0699v1

      The basic problem is that the findings of large-scale research collaborations like this are probably directed by government bureaucrats sitting in offices of federal funding agencies.

      With kind regards,
      Oliver K. Manuel

      • I counted 499 coauthors !

        The funding sources acknowledged:

        U.S. Department of Energy and National Science Foundation; the Italian Istituto Nazionale di Fisica Nucleare; the Ministry of Education, Culture, Sports, Science and Technology of Japan; the Natural Sciences and Engineering Research Council of Canada; the National Science Council of the Republic of China; the Swiss National Science Foundation; the A.P. Sloan Foundation; the Bundesministerium für Bildung und Forschung, Germany; the Korean World Class University Program, the National Research Foundation of Korea; the Science and Technology Facilities Council and the Royal Society, UK; the Institut National de Physique Nucleaire et Physique des Particules/CNRS; the Russian Foundation for Basic Research; the Ministerio de Ciencia e Innovación, and Programa Consolider-Ingenio 2010, Spain; the Slovak R&D Agency and the Academy of Finland.

      • To See How Public $$$$ => Consensus Science

        Study the list of names and funding sources for the 499 coauthors:

        http://arxiv.org/pdf/1104.0699v1

        Or send a request for the 12-page list compiled yesterday in my analysis of the sociology of consensus science: omatumr@yahoo.com

        [I won’t be able to reply until Monday.]

  2. Leinen’s essay is insightful, and increasingly cogent as our data expand at increasing rates. There is no single perfect remedy, but I believe two principles will be helpful. The first is the need for important results to be independently replicated, perhaps even more than once. The second is the need for error correction when previous conclusions must be discarded or modified. The difficulties arise when results are conflicting, but ultimately one hopes that the conclusions that prevail are those that best reflect the reality of nature.

    • Perhaps most importantly is to follow Popper’s insight that while we may say that our knowledge is and always will be conjectural in nature, and therefore subject to change, we can be certain when evidence disproves our propositions. Leinen is struggling with the same issues that philosophers have been struggling with for over 2000 years. From Plato and Aristotle to Hume and Kant….

    • Fred,
      Does this mean you are ready to free yourself from AGW dogma and join the rationalists in skeptically critiquing AGW claims?

    • One can argue that the fact that different climate models disagree is a failure of replication. If so then all modeling results should be rejected as untrustworthy.

      • David,

        ‘One can argue that the fact that different climate models disagree is a failure of replication.’

        Disagree on what? Does disagree mean that 0% agreement? 25% agreement? How about some specific levels of attainment here?

        Most of the models used (GCM’s) are parameterized by different groups to optimize the ability to produce specific climate phenomena. That is, one group is interested in black carbon feedbacks on albedo, therefore they iterate their model runs to reproduce phenomena related to albedo changes in response to soot from the historical record. Another group would do the same for ocean currents.

        That means, when compared to one another, things are going to look differently because different physical processes are highlighted as ‘important’. The model can’t get everything ‘right’, which could be an issue, but it certainly does not represent a ‘failure of replication’.

        As an example, I can calculate the electronic energy levels of C60 fullerene (bucky balls) molecules with a number of methods. Each method is optimized to match a specific behavior of the molecule. One may get non-linear optical parameters correct while another is optimized for electron transport. The values for the energies of the states, however, do not agree 100%. Does that make the model for fullerene and/or quantum mechanics ‘untrustworthy’?

        In my opinion, no.

      • I am referring to the wide range of global temp projection under standard scenarios, not to mention the contradictory regional precip projections. There is actually a literature on this problem, especially the lack of convergence after over 20 years of trying. Perhaps you are not aware of this problem. The projections range from benign to catastrophic. They are why we are all here debating.

  3. This is equally true of a wide array of fields of human endeavour. The complexities of the basic sciences underpinning medicine, for example, are way beyond the capacity of any individual to comprehend. Moreover, the complexity extends from processes occurring at a molecular level through to behaviours of large populations with added layers emanating from the economic and administrative aspects of service delivery, determination of priorities for intervention, and indeed the competition between health care provision and the provision of other goods.

  4. This isn’t so new.

    Douglas Hofstadter’s Gödel Escher Bach took the concept a level further and presented individuals as ants — unintelligent ones at that — in an intelligent ant colony. (He even had an anteater who doctored the colony by eating ants.)

    Our difference is we know we’re in a colony that has behaviors, ‘thinking-processes’ and actions orders of magnitude beyond our individual scale, and some of us flatter ourselves that we know what’s best for the colony, or can anticipate its behaviors and thoughts.

    We’re pretty gullible that way.

    However, we can build anteater organizations and networks, which if large enough and suitably designed, might attempt such anticipation, such meta-psychoanalyses, like the Dwarf to Micromegas; we’ve tried it with governments of nations and corporations, institutions of higher learning and thinktanks.

    We’ve even learned a little about the limitations and uses of such efforts, which often take on an independent life and will of their own.

    And likewise if we can build mixed-metaphor anteater-networks, why can’t we build up such compound beasts to address any complex and overarching problem space like climate?

    We’re not guaranteed of success every time, or the majority of the time, but what of it? We know how to measure the success or failure of corporations and governments.

    Better one eye than none, to further mix the metaphor.

    In the land of the blind, the one-eyed man is king. We need an eye into the complex that none of us alone can see.

    It’s readily apparent our climate eye needs some optometry, but isn’t that just a matter of refinement?

    In the meantime, it’s merely Dunning-Kruger effect run amok to believe one’s own secret formula, pet theory, homebrewed method or self-confirmed invention can compete on in the problem space of such complex issues.

  5. There is no Supreme Court of Science; there never has been. It is simply not necessary to understand all the complex details of things theoretical. The Supreme Court of Science is the measured, preferably experimental, data. “To the solid ground of Nature; Trusts the mind that builds for aye” William Wordsworth.

    When we have enough hard, measured data, we will be able to demonstrate that CAGW is a load of hogwash; it always was, still is, and always will be just a hoax.

  6. It has been my experience in life that when faced with complex situations or problems, the most logical way to proceed is to break the complex into manageable components. By building on solving the simple within the complex it is quite easy to reach a solution or at least an understanding.

    The climate science community seems bent on maintaining a cloak of complexity around themselves, using the obvious complexity of the Earth’s climate system as a shield from answering the simple questions which are necessary to be answered in order to prove their theory on global warming. The entire global warming narrative is based on silly unscientific practices such as model generated scenarios, from which springs forth countless academic studies and projections which generate further studies and projections and on and on it goes. All of this based on computer generated assumptions of what the climate will do based upon increased CO2. Why do we have these numerous computer generated scenarios which feed this beast of scientific gluttony?

    What is the climate sensitivity for a doubling of CO2 ?

    • Jerry a very sensible approach.

      In my work i am often confronted with exceptionally complecated problems so i break the issue down into component parts and identify the crucial aspects- i.e. the critical parts of the ‘problem’ that have the largest effect or underpin the whole problem.

      Most complex issues can be broken down this way with sufficient training. I think it is disingenuous to suggest (if that was indeed the intent) that cAGW and by extension climate science, being so complex with multiple interrelated and interconnected aspects is not open to full comprehension to individuals and must instead rely on a ‘consensus’ approach.

      To put it another way, the cAGW is a very complex issue- but- if i were to for example, to determine that clouds and water do not infact amplify the co2 ghg, then i wouldn’t need to know all the other aspects of the theory/science to have exceptionally high confidence that the theory was incorrect.

      • TimTheToolMan

        But on the other hand if you could see that clouds and water did in fact amplify the effect of CO2 as a GHG for the period studied, then you still need to understand why that occurred so that you can be sure its not going to change in the future and thus you end up with a broken hypothesis.

        In a complex system such as the climate, having confidence in a particular result doesn’t mean you can ignore the other complexity because your result inevitably exists within some context that itself may be subject to change.

      • TimTheToolMan,

        “… that clouds and water did in fact amplify the effect of CO2 as a GHG …” needs to be validated just like those data 390W/m2 Earth Surface Radiation, 324W/m2 GHG Back Radiation, 35W/m2 CO2 radiation contribution …

      • No- if that were the case you’d move ontop the next crucial aspect- climate sensitivity. Rinse and repeat.

      • Lab,

        Ever think this planet is drying up? Loosing a great deal of vapor to space? What was this planet like 4.5 billion year ago? What stopped the mass from leaving the planet from centrifugal force? With the vastness of the abundance of sand, how was that created?

        We were actually a water world with vast underwater volcanoes with the volume ocean water 10 times the current ocean levels.
        When did land mass start to appear?

      • Joe,

        Think of Gravity.

      • Sam,

        If you had a vast amount of pressure and stored gases to generate many cracks and volcanoes, that would generate a vast amount of debris and mass to hold water to the planets surface.
        This is molten rock mixing with a vastly greater amount of ice and water.

      • Sam,

        We do have high level water vapor.
        Hmmm…I wonder if the gravity is as strong there as low laying clouds?

      • Joe,

        Volcanoe eruption pressure lack of substained energy (unlike some special design rockets) to propel H2O beyond the stratosphere.

      • interesting.

        Thinking about it i wouldn’t be suprised if we lost material to space, i also wouldn’t be suprised if we gained some while falling through space.

        I’d imagine we’re loosing more than we accumulate though- though very very gradually. Hmm, i’ll have to have a think about this one.

      • munkey,

        “Hmm, i’ll have to have a think about this one.”

        Don’t go too deep that it becomes another scaremonger like CO2. H2O is tied down by gravity.

      • Yes of course, i’m not forgetting gravity- but gravity will allow particles to escape. We may be talking ppm over century’s, but in the lifetime of a planet- is it significant?

        Just a mental exercise, nothing serious.

      • So, Sam,

        What is the date of the oldest salt mine and what is the land height of that mine?
        Simple exercise.

      • Partciles traveling at the speed of light, of course will not come under the influence of gravity, H2O is different,millions of times traveling slower.

      • The only substance that has escape velocity at Earth’s surface temperatures is hydrogen. So we’re losing mass, but it’s very, very slowly. Whenever free hydrogen is generated, it’s a race between escaping the atmosphere and reacting with something. Hydrogen is highly reactive, though, so usually escape loses.

        Meanwhile, stuff hits us all the time as we hurl through space. The net result is mass gain.

      • QB;
        That’s on average. With Brownian collisions and motion, there’s always a huge spread of energies and velocities between different molecules. Some will be >escape velocity at that tail of the distribution.

      • TimTheToolMan

        But climate sensitivity is the combination of the whole gamut of complexity so you’re back to square one. If you believe you have some aspect nailed down because you’re really happy with the analysis then thats actually of much less worth in the larger context.

      • Climate sensitivity was just an off-the cuff example.

        Perhaps a better one would be temp data contamination, or the original one on clouds.

        I fully accept that the issue is incredibly complex, however there are critical aspects of the theory that if understood in isolation can have dramatic effects on the theory as a whole- this is what i’m reffering to.

  7. By contrast, until quite recently the complete justification for even the most complex scientific facts could be understood by a single person.
    A single person could understand complex scientific facts before now and could not after Now. Does anyone really believe this?
    Science is no longer so simple; many important scientific facts now have justifications that are beyond the comprehension of a single person.
    A single person could understand complex scientific facts before now and could not after Now. Does anyone really believe this?
    We do now know more about a lot of things, but they have not become more complex. Our knowledge is more complex and we may have a better understanding about what we do not know, but we did not just get Stupid.

  8. I’m afraid that I don’t buy this as relevant to climate change. All scientific disciplines have been this way for as far back as I can count. Human knowledge has always been hierarchically structured and fractured.

    It also seems to me that this proposal of fractured science is being offered as an excuse for some of the inadequate quality control that is being discussed as regards climate change.

    For example you say: “Because of the complexity of many climate studies, peer review is typically cursory in its ability to uncover methodological or logical problems.”

    I don’t buy it. The peer review process should always be able to uncover methodological or logical problems; complexity is not an excuse. If flaws are not being uncovered by the peer review process, then either the reviewers need to be investing more time in their reviews, or more people need to be added to the review list.

    • Hear hear. The Peer Review Process seems to be moot other than to promote the idea or validate the funding. I’ve worked with Quality Assurance teams nearly all of my career and their job is to BREAK our work. When they have failed to break the system, the system is ready for the public.

      The AGW Peer Review is sort of like Microsoft. “Are you saying we purposely misled you about what we advertised!? Would thousands of professionals who were paid to do this lie to you? That’s not an error or bug, that is a feature, just ask one of our experts. Deal with it until our next Release, Click.”

    • Perhaps the problem is not that the peer review process is inadequate, which it probably is, but that too much credit is given to the process. Peer review is not and has never been the final arbitration of scientific ‘truth.’ A positive peer review merely indicates a paper is worth publishing and that no silly errors were immediately obvious. It is not until the paper is reviewed by the field at large that its true value is determined. And at that, its value is provisional, subject to modification or falsification by other studies.

      • I suspect, based on the experience in my own niche of the peer review process and looking out towards others in disciplines both more and less rigorous, that there are cultural differences between different disciplines that determine how rigorous their peer review process is.

        Peer Review in fields such as mathematics in which people attempt to understand things as complex as the Riemann hypothesis are outlandishly rigorous from my perspective. I have heard that such reviews take months to write. It’s perhaps easier to write a paper than to review one in some cases.

        In my field, I see a mix of rigor. Some reviews are cursory, some less so, but all of them are less rigorous than the tales I’ve heard about publishing in Math fields.

        Therefore I am forced to conclude by induction that there are probably other fields with even less rigor than mine.

      • Climate community peer review process has the least rigorous process – did not even check data in the papers/articles with the consensus and ruled out any non-consensus, sad.

    • The indictment of the Peer Review process in Climate work is not that it is “cursory” or rigorous, but that it is inconsistent and not even handed. There are cases where important work is suppressed or delayed; yet simultaneously trash somehow gets published. That this is no accident is supported by the Climategate emails.

  9. JC, this is wrong in so many ways it is difficult to decide where to start.

    First let’s start with Hayek. Your quote actually incomplete and has some ‘hidden’ knowledge.

    “. . . which all the separate individuals possess. The economic problem of society is thus not merely a problem of how to allocate ‘given’ resources –if ‘given’ is taken to mean given to a single mind which deliberately solves the problem set by these ‘data’. It is rather a problem of how to secure the best use of resources known to any of the members of society, for ends whose relative importance only these individuals know. Or to put it breifly . . .”

    This in no way relates to science. The best use of resources known to any participant in the scientific process is to make that resource known, clear, and able to reproduce for independent testing.

    If you read on Hayek goes into ‘who’ is doing the planning of the said resources. The AGW model is most certainly following a Central Planning methodology which assumes the Central Authority to Coerce an outcome where a Distributive model allows any individual the ability to organize the available knowledge to Predict an outcome.

    :) Facts need no justification.

    Science has never been simple nor complex, these are subjective terms, but it has been disciplined except when Anthropogenic Authority interferes. (AGW, Piltdown Man)

    Scientific Consensus is an oxymoron. Consensus is a majority of opinion and Science seeks to render Opinion moot. The only Authority in Science are the Laws of Physics.

  10. ian (not the ash)

    In the not too distant past when I was a vociferous CAGW campaigner (among many other activities I was an active member of a group that toured our area presenting ‘Australia’s Inconvenient Truth’) there were certain tenets that were routinely asserted at ‘climate science’ presentations:

    1. The science is settled and debate is obstructionary,
    2. It’s worse than we thought, followed by the obligatary addendum…
    3. …but it’s not too late,
    4. Current technologies are capable of supplying the world with near zero emission energy needs, if the political will was there
    5. Bone-up on the science at the next ‘An Inconvenient Truth’ training session…

    I felt absolutely no compulsion to question the scientific bases of these claims (except no. 3 … surely by now it must be too late??) because in truth, I never bothered. Indeed, I’m sure many active in the movement probably could’nt tell the difference between the PDO and a bad case of BO. While still a scientific ignoramus, I have now discovered enough to make me deeply embarrassed about my former exhortations.

    • Science is never Settled

    • Ish,

      Welcome to the dark side. It is like the nuclear fear all over again. Anti-nukes still selectively quoting their mis-information, avoiding new studies that contradict their tribal mentality. There is an interesting puzzle that is pretty fascinating that they are missing.

      The climate puzzles are neat too, but it is like questioning a religion. CO2 warming should be mainly in the upper troposphere and the poles. The poles are so whacked out in relation to each other it is hilarious, but it is “consistent with the models”. The TTS temperature has a trend of 0.005 per decade. In one paper I saw a model estimate that was “=/- 0.19 W/m^2” accurate being compared to a real world test instrument with an accuracy of +/- 7 W/m^2, and they are looking for a few terrajoules of missing heat? Nonlinear forcing is estimated at less than 10% based on a couple of modeled slabs representing oceans?

      There is just a touch of uncertainty, but you are a heretic to bring it up. Is there any wonder so many engineers, statisticians and meteorologists are skeptical?

    • Ian,

      You are only one in the crowd of former CAGW believers. I even voted for Al Gore for President!

      I am still an environmentalist, strongly believing that we should minimize pollution and wasteful production, but I am most disillusioned with the politicians who twisted government science into a tool of government propaganda.

    • As a former believer, I welcome you to the rational side of the street.
      The liberation you will experience by being able to deal with things factually is wonderful.

  11. JC or AGW Expert,

    How is it that there are thousands of climate experts and entire heavily funded organizations dedicated to Climate and no one has thought to put the dozens, if not hundreds (thousands?), of Climate hypotheses into an organized Theory so they can be proven by independent testing?

    Not peer reviewed. Critic Reviewed. If a Theory is sound, the most ardent and capable skeptics’ failure to falsify it will make it rock solid.

    We’re not talking micro economics which requires one to accept the price that is available. We’re talking about the Scientific Method which certainly has been followed to create such ardent believers, correct?

    All the Methods and Conclusions have been clearly recorded so they can be repeated by friendly colleagues (sometimes known as Peers), right? And once they have verified their colleagues’ incredible conclusions, they have made those methods and conclusions available to all, especially those capable of falsifying them for extra assurance no errors were made, true?

    If this has not been done then the level of effort to do so should be minimal if it is in fact Science that has been practiced to produce so many studies make so many believers. If the number of believers is a result of Public Relations, Trust, Consensus, Authority, Coercion, and/or Fad the AGW Experts have their work cut out – from under them and similar careers will suffer.

    All scientific institutes are going to be terribly hurt by this fraud. It displeases me to see this, but the fact that those in non-climate related sciences have remained so silent when what has been produced and transpired is so egregious to sound science it is akin to the you name it moderates who silently protect the reckless fanatics who wasted billions in research and advocate trillions in wealth redistribution.

    Good luck, you earned it.

    • The End is FAR,

      “… but the fact that those in non-climate related sciences have remained so silent when what has been produced and transpired is so egregious to sound science it is akin to the you name it moderates who silently protect the reckless fanatics who wasted billions in research and advocate trillions in wealth redistribution.”

      The concensus will lodge propaganda with fierce attacks on these real scientists. Results – got fired or silenced. Did you not see those spoken truth got fired professors and even the former Australian PM got fired. A lot of big company CEOs will not dare say anything against these concensus to stay in the market to prevent fierced concensus media propaganda attacks.

      • AGW is an economic and scientific bubble. It is close to bursting or more proper hyper-deflation. There are two primary factors. First it has no actual value, it is completely speculative. Sure it has value to those that are selling it, but they are selling fear and that fear has rapidly declined as the main players erred in piling on too much BS. They became overconfident in their ability to sway the easily coerced and their ‘Theory’ became a prisoner to PR(good marketing of a bad product), Authority(who says?), Credibility(useless), Scientific Consensus (oxymoron), and Fad.

        They cannot produce a sound Theory because what they have produced to date does not pass muster. It is highly contradictory, it can’t be tested, the conclusions of some studies rely on unconfirmed conclusions of other studies, and it only stands under very special circumstances where some phenomena must be ignored like Convection (it cannot predict general outcomes, only very restricted ones). It is a long train of mistakes and excuses that cannot stand a month of Critical Review.

        Second, the supply of money has been exhausted. One may say this is false since the AGW community is still being funded, but with what? It is all borrowed (unconstitutionally stolen IMO) and inflated money and becoming less valuable every day. Many people predict that our economy is going to collapse, but that is a false paradigm, it is the Governance that Centrally Plans our collective economies that will collapse. The ability to continue inflating this ‘fear’ has been exhausted. Just like thermodynamics economies are in a constant state of flux, inflating or deflating, and when an energy supply exhausts, there is no ability to inflate nor keep at a constant volume, it will deflate rapidly. With gas expected to hit $6/gal with inflation, people will lose all their patience for the fossil fuel deniers.

        An economy is simply “The Management of One’s Household”. The National economy is nothing but the sum of all the Local Economies. Many people (enough) have recognized that relying on a few people in Washington DC to Centrally Plan highly distributive and variable Economies (Households) can no longer be burdened nor tolerated. After the restoration of Local Governance that will result from this macro collapse, Science will also take a big hit. (The collapse is imminent, our money is devaluing so we have to borrow twice as much as our ability to repay it is declining) The trust in the science industry that was destroyed by crony Socialism (is there another kind?) and crony Capitalism (Socialism/Corporatism) will take a great deal of effort and sacrifice to rebuild.

        Sure this is itself speculation, other outcomes could happen, but the scientific fraud/exposure and economic events are already underway. Grants for Universities, Laboratories, etc from Foundations, Gov’t, and individuals will be very limited.

  12. Since some discussants have referred to Hayek, and an article of his was cited in Judy’s post, it would be good for Judy to correct the text of her second paragraph, where she wrote “the economist Frederick”, to “the economist Friedrich von Hayek” (Frederick is an English rendition of his German name).

  13. How can we be sure the results of large-scale collaborations or computing projects are reliable?

    Dr. Curry: It’s a tricky question, but in regards to our blog subject, climate scientists could make falsifiable predictions about future climate, then live or die by the results.

    But we both know that they won’t do that. They’ll take credit if climate goes their way and they will make excuses if it doesn’t.

    When climate scientists have a track record comparable to medical doctors and infectious diseases, I’ll be happy to take them more seriously.

  14. I think that the very fact of climate science intersecting crypto-knowledge in anyway at all is an incredible indictment against its credibility.

  15. I am wary of these it-is-too-complicated-to-understand complaints about climate science. It is very simple. Doubling CO2 is equivalent to raising the solar input by 1% and sustaining it. Would anyone argue that warming would not occur if that happened, or is that too complicated too?
    It is the unprecedented size of the forcing that makes this a special case. Note that the sun has not varied by as much as half a percent in recent millennia, and even those small variations resulted in the LIA and probably MWP.

    • andrew adams

      Indeeed – the basic scientific arguments in favour of AGW are pretty straightforward, it’s the skeptics who tend to use the “it’s too complicated” argument.

      • aa,
        No, the skeptics simply point that no catastrophe is taking place.
        Believers keep pretending there is.
        It is the believers who invoke complicated high positive feedbacks for which there is no evidence.
        The skeptics merely ask, where is the evidence of the strong feedbacks?

      • andrew adams

        hunter,

        Are you saying that skeptics never use the argument that our climate system is too complicated for us to have sufficient understanding to attribute the recent warming to CO2?

        I’m not sure why you bring up the subject of feedbacks, but there is a great deal of research based on multiple lines of enquiry which supports the contention that they are positive and that climate sensitivity is in the range of 1.5-4.5C
        If you want evidence for positive feedbacks look around you – you will notice that we are no longer in an ice age.
        Claiming there is “no evidence” whilst wilfuly ignoring a large body of scientific research is, well, denial.

    • Ah there’s the rub. The alleged “unprecedented size of the forcing.”

      Your certainty would be more convincing if there was more certainty about that, but there isn’t.

      And without that…

      • Al,
        There you go asking for things that are too complex for youthe believers to explain. You need to have faith, like they do.

      • Well hunter, the younger you are, the more unprecedented everything is… Seems things are so unprecedented these days that nothing ever happened before last Tuesday.

        But rumor has it that something called ice core data has a whole bunch of hockey sticks goings every which way, so something might have happened before.

      • Al,
        Our AGW promoters do not like appreciate your speculating, no not one single bit.

      • I say again, CO2 doubling far exceeds anything the sun has done in the last millennium. I have given numbers to show it in other replies here. Let’s compare 3.7 W/m2 from CO2 doubling to, at most, 0.5 W/m2 solar forcing that caused the little ice age or possibly the MWP. Doesn’t it seem CO2 should be noteworthy given that?

    • Jim D,

      “Doubling CO2 is equivalent to raising the solar input by 1% and sustaining it.”

      You have calculations?

      • Jim D,

        So now you have disproved K&T’s 1997 Global Annual Mean Radiation Budget of 342W/m2 sun radiation against 35W/m2 CO2’s Back Radiation which is more than 10% not yours 1%.

        Climate community has the habit of putting out any numbers with the support of their models, I suppose.

      • 3.7 W/m2 is the forcing from doubling CO2, 3.4 W/m2 is the forcing from increasing the sun’s strength by 1%.

      • Your modeling is getting weird,” doubling CO2, 3.4 W/m2 is the forcing from increasing the sun’s strength by 1%.”?

      • I’ll let someone else explain to you why 1% of 340 is 3.4. I’m not going to even try.

      • Increase the sun’s power by 1% is weird logic. No body can explain it except by yourself.

      • OK. Now I understand how alarmist logic works, twisting physics and radiations, LOL.

      • That 1% still has no basis, unless you can account for it.

      • This was used as a comparison with what doubling CO2 does to the energy balance. It is easier to imagine the sun increasing by 1% than CO2 doubling, but they have the same effect on forcing the climate to change.

      • Jim D,

        Thats is apparently your conclusion not how you arrived at that 1% which more details would help to understand.

      • OK, 3.7 W/m2 is the forcing from doubling CO2 (before you argue, even Lindzen agrees with that). What percentage of solar radiation is that? Solar radiation at the top of the atmosphere is 342 W/m2, so 1% about matches CO2 forcing. That is where 1% came from. If the sun faded by 1% it would cancel the CO2 effect, if that makes you happier.

      • Jim D,

        Don’t mind other skeptics, how did you get 3.7W/m2 by doubling CO2?

      • I would use a science text to get 3.7 W/m2. Where would you look? Do you have an alternative number supported by anyone?

      • Jim D,

        Where do you find a science text that substantiated 3.7W/m2? Or just a spurious number in that text without science?

      • Assume the IPCC’s 3.7 W/m2 for CO2 is correct. The sun averages 342 W/m2. We know from Stefan-Boltzman that energy radiates as the forth power of the temperature. So

        342 = n T1^4 , where n is a constant and T1 is todays temperature, say 288K

        342 = n 288^4
        n = 342 / 288^4
        n = 4.97114E-08
        and with co2 doubling you get
        342 + 3.4 = 4.97114E-08 T2^4
        T2 = (345.4 / 4.97114E-08) ^ -1/4
        T2 = 288.7 K
        Change = 288.7-288 = .7 C

        So, with a doubling of CO2, you will get .7 C of warming.

      • CO2 levels are currently about 400 ppm and increasing about 2 ppm. So, to double to 800 ppm will take 400/2 = 200 years. Even if rates double as China, India and the rest of the world industrialize, we are looking at about 150 years for temperatures to rise a further 0.7 C due to CO2. I have bigger temperature changes in my backyard hour to hour.

      • ferd,

        You are probably right except alarmist said it will have catastrophy effect on sea water levels due to melting of polar caps, polar bears will be distinction, ocean will be acidified, corals will be bleached, typhoons/hurricanes/tornados (I made it up?) will increase, flooding increase, more torrential rains, almost anything bad with alarmists’ lost of magnitude senses!

        K&T’s 1997 Global Annual Mean Radiation Budget said CO2’s contribution is 32W/m2 (at 380ppm CO2 at that time), now (390ppm) doubling (780ppm) that only contributes 3.7W/m2 additional radiation seems weird and if true than we never have to worry about concentration of CO2 in atmosphere as doubling of 780ppm to 1560ppm only contribute another 0.4 W/m2 and then doubling of 1560ppm to 3120ppm only adds another 0.05W/m2 … scientifically, why bother care about the alarmists claims?

      • It depends upon human behavior going forward. I’m in the oil industry. Every time people have suggested we are running out, we find more. We find more, the world’s population buys more. Will that behavior persist?

        Past human behavior has led to the anthropogenic component doubling every 30 years. That component is current ~112 ppm (112 +280 = 392,) so in about ~2040 past human behavior indicates the anthropogenic component could be ~224. 280 + 224 is 504 ppm, which is 56 ppm short of doubling CO2: 560 ppm.

      • ferd,

        Thanks for the calculation but I was really after that 3.7 W/m2 from CO2. How do they manage to get it?

      • it is calculated by using smoke and mirrors (computer models).

        The argument is made time and again that we don’t a means verify the GHG theory by observation because there is only one earth, thus we must rely on computer models. This argument is just so much hot air.

        We have a solar system full of planets and moons, many of which have atmospheres. The simplest and most direct method to validate the GHG theory is to observe the effect of GHG on other planets.

        We know from studying that atmosphere of other planets that once we adjust for distance from the sun, each planet shows the same temperatue at the the same pressure.

        In other words, it is the thickness of the atmosphere that determines the temperature, not the composition.

        Venus and Mars both have CO2 atmospheres. Earth has an N2O2 atmpsphere and Jupiter has a H2 atmosphere. Mars in particular should be warmer than it is due to CO2 (GHG effect) but it is not.

        The surface pressure of Earth is 160 times greater than Mars. The Martian atmosphere is 95% CO2. Earth’s atmosphere is 0.038% CO2 – which is 2500 times less than on Mars.

        So, at the surface of each planet, Mars has 2500/160 = 16 times more CO2 per cubic meter of atmosphere than does earth. This should have heated the planet much more that it has if the GHG theory is correct.

        It had been assumed that because Venus was hot, it must be due to the GOs atmosphere, without considering that Venus has an extremetly thick atmosphere. The surface pressure is 90 times that of earth. On earth that is the pressure we find 90 x 30 = 2700 feet under the ocean!

        When you look at Venus at altitude, such that the atmospheric pressure is the same as Earth, the temperature as compared to Earth is the same as their relative distances from the Sun.

        This is impossible if the GHG theory is correct and is grounds for falsification of the GHG theory. It is widely accepted that falsification of the GHG theory is currently not possible by only looking at the Earth, which is the rational for models.

      • A one-dimensional radiative transfer model is sufficient. It comes from known spectroscopic properties of CO2 in a known atmospheric profile. Very basic science.

      • Jim D,

        “A one-dimensional radiative transfer model is sufficient. It comes from known spectroscopic properties of CO2 in a known atmospheric profile. Very basic science.”

        Show me that article/research has CO2 substantiated 3.7W/m2 radiation. You can manipulate models with data
        to provide desired number – pseudoscience.

      • We went through the exercise with MODTRAN before.

      • Jim D,

        “We went through the exercise with MODTRAN before.”

        Yes, that model. I tend to not trust models which usually have a lot of assumptions with manupilated outcome. But if you have the credible details of the logarithms how it calculated the CO2’s 3.7W/m2, I will perhaps change my mind to accept Modtran as an exception.

      • MODTRAN integrates over all the spectral lines of atmospheric gases for a given atmospheric profile of temperature and GHG amounts. Its database is thousands of spectral lines that are measurable and verifiable by spectroscopy, but each line has a simple function describing its absorption/emission strength.

      • Jim D,

        “MODTRAN integrates over all the spectral lines of atmospheric gases for a given atmospheric profile of temperature and GHG amounts. Its database is thousands of spectral lines that are measurable and verifiable by spectroscopy, but each line has a simple function describing its absorption/emission strength.”

        I am only interested in CO2. I am ready to be convinced that CO2 has such effect warrant the whole world’s sacrifications to abate CO2 emissions. Get those CO2 spectral lines and show me that 3.7W/m2 doubling effect or show me K&T’s 32 W/m2 back radiation spectral line calculation at 1997 when the CO2 has 380ppm.

      • You can do that with the online MODTRAN, as I showed you before. Double CO2, and see what happens to outgoing longwave at the tropopause. It is reduced by about 3.7 W/m2 depending on which sounding you chose. MODTRAN shows the spectral response too, so you can see most of the change is near 15 microns. Reduction of outgoing has the same forcing effect as increase of incoming.

      • Jim D,

        You believe in MODTRAN and I doubt very much about MODTRAN. I am not getting anywhere, never mind, I gave up these weird models.

      • Almost correct. Take albedo into account, which multiplies 342 by 0.7 and you get 255 K. Then adding 3.7, you get about 1 C increase, which is the no-feedback response. Add to that the effect of having 7% more water vapor as a result of that 1 degree, and its recurring feedback on further increases, then you get nearer the 3 C increase in simplified terms.

      • No, what I showed was that a very simple back of the napkin calculation provides a result of 0.7 C due to CO2 over the next 150 years.

        As soon as you start adding in the correcting factors as you propose, you must start to correct for all the known problems, not just the ones that increase the result.

        For example, the calculation starts by dividing TSI/4 to end up with “sun averages 342 W/m2”.

        This in itself if a BS number, because this is not how the sun works. It shines only on one side of the earth, and then most brightly only upon one spot.

        This affects Stefan-Boltzman, because we are dealing with a power function. You can’t move the average inside the power function as is done by climate science without invalidating the result from the get go.

        eg: AVG (x^1/4) is not equal (AVG(x))^1/4

        where x = t1,t2,…tn , the temperature at different places all over the earth.

        The correct solution is to calculate Stefan-Boltzman pointwise and then integrate this over the surface of the earth.

        A second problem is the correction for the atmosphere. This is a huge problem because the effects of convection and clouds are poorly understood and the numbers are largely guesswork.

        So, knowing that be basic first step of dividing TSI by 4 is incorrect, adding multiple factors, many of which are simply guesswork cannot hope to increase the accuracy of the result except by accident. The more factors you add that are not well understood, the greater the uncertainty in the result.

      • Ferd, you can keep it in radiation units or convert it to temperature as you did at first before saying it was wrong. CO2 has a substantial effect on the balance at the top of the atmosphere that is comparable with a 1% solar increase.

      • Then adding 3.7, you get about 1 C increase, which is the no-feedback response. Add to that the effect of having 7% more water vapor as a result of that 1 degree…

        Except that you first need to get the 1C increase before you can get the 7% more water vapour.

        As the temperature starts creeping up towards that 1C increase, both convection and evaporation start increasing and, as both of these processes require energy (evaporation requires yet more due to latent heat), a good part of your original 3.7W is used up long before the temperature increases very much.

        Bear in mind that this 3.7W will still leave the surface one way or the other, just not all by way of increased radiation.

        Also, you’re assuming a steady-state average of 3.7W. To get the true picture you have to take into account things like the thermal capacity of the surface material, and integrate the figures over the length of day (and night) in order to get the true average temperature change resulting from this.

      • Also, can you think of a reason why your hypothetical 7% increase in water vapour should not result in increased cloud cover?

      • Peter317, 1 C is the increase in effective radiative temperature required to balance the budget. You may ask how does the atmosphere adjust convectively to achieve that? It maintains its lapse rate, which is determined mostly by convection, which translates that 1 C as a fairly uniform change through the profile to the surface. If the ocean is 1 C warmer, it has 7% more water vapor in equilibrium at its surface. The atmosphere redistributes this through the troposphere increasing the greenhouse effect further but keeping RH near constant. If you know of a mechanism by which water vapor won’t increase above a warmed ocean surface that would be interesting to see. If RH stays the same, cloud cover should also stay the same.

      • Jim, if the lapse rate is maintained at the same uniform profile, a 1C increase at the TOA would mean a 1C increase at the surface.
        But, because of convection and evaporation, a lot more than a 3.7W increase would be required to increase the surface temp by 1C.
        So where does the extra energy come from?

        If you know of a mechanism by which water vapor won’t increase above a warmed ocean surface that would be interesting to see

        My point is, you first have to get the temperature increase before you can get the vapour increase.

        And cloud formation is not dependent on RH.

      • Jim D,

        The latent heat of condensation of water in the temperature range from −40 °C to 40 °C is approximated by the following empirical cubic function:
        Lwater(T) = − 0.0000614342T3 + 0.00158927T2 − 2.36418T + 2500.79

        At 20°C, Latent heat of evaporation is =2500.79+0.00158927x20x20-2.36418×20-0.0000614342x20x20x20=2454KJ/kg

        Evaporation rate of water at 20°C=3.7/(2454×1000)=1.51×10^-6kg/s/m2

        At 19°C, Latent heat of evaporation is =2500.79+0.00158927x19x19-2.36418×19-0.0000614342x19x19x19=2456.0229KJ/kg

        Evaporation rate of water at 19°C=342/(2456.0229×1000)=0.13925×10^-3kg/s/m2

        % water vapor content increase due to 1°C rise in temperature= 1.51/139.25=1.08%

        Jim, verify your 7% increase.

      • Peter317, You have to remember the imbalance is a flux of energy. It is a persistent heating, and yes, it has to heat the ocean surface layers too, so depending how deeply you heat the ocean it might take a few years given the heat capacity and thermal inertia.
        Regarding cloud cover, there is no reason cloud cover should increase with higher temperatures. Convective areas should stay the same.
        SamNC, 7% comes from the Clausius-Clapeyron equilibrium over a water surface. Raise the water surface temperature by 1 C, and the equilibrium water vapor increases by 7%.

      • Jim D,

        ” 7% comes from the Clausius-Clapeyron equilibrium over a water surface. Raise the water surface temperature by 1 C, and the equilibrium water vapor increases by 7%.”

        I am not an expert with Clausius-Clapeyron equilibrium, can you link to that paper or calculation that showed 7% increase in atmospheric water vapor with 1 deg C temeprature rise? My common sense seems telling me is unbelievable. If thats true, the alarmists might have a point of increase flooding and more rain … whatever disasters they thought of were linked to with 3 degC rise.

        Jim D, please do calculate or link me to that Clausius-Clapeyron equilibrium showing 7% water vapor increase.

      • Jim, you’re confusing cause and effect. I’m well aware that we’re talking about energy flux, but as we’re also talking about equilibrium conditions, it’s immaterial how long the surface takes to warm.
        I’ll put it another way.
        Let’s assume that the surface – for whatever reason – is 1C warmer. Now it must be radiating 3.7W/m2 more. But, being 1C warmer, there must also be more convection and evaporation, which both require energy to happen. So the total energy flux from the surface must be more than 3.7W. As the energy flux to the surface is still 3.7W, equilibrium does not exist and so the surface must get cooler.

      • Sam,
        You can checked, how the partial pressure of water that corresponds to 100% humidity depends on temperature from this calculator

        http://www.efunda.com/materials/water/steamtable_sat.cfm

        The partial pressure of water that I mentioned above is the same pressure as the Absolute Pressure given by this calculator. The moisture content of the air at a fixed relative humidity grows proportionally to this value with increasing temperature.

      • Peter,

        Let’s assume that the surface – for whatever reason – is 1C warmer. Now it must be radiating 3.7W/m2 more. But, being 1C warmer, there must also be more convection and evaporation, which both require energy to happen. So the total energy flux from the surface must be more than 3.7W.

        What you write is true for the total gross flux from the surface, but there is also down-welling radiation that increases with increasing temperature of the atmosphere. Thus the net flux does not change as much, not even nearly as much.

      • SamNC, the C-C formula is de/e=dT*Lv/(Rv*T*T)
        where e is that saturated vapor pressure, so de/e is its relative increase, dT is the temperature increase, Lv is the latent heat of vaporization=2.5e6 J/kg, Rv is the gas constant for vapor=460 J/kg/K, and T is the temperature in K.

      • Everybody seems to be ASSuming that as you raise the surface temperature, the relative humidity stays constant. There’s no reason for that to be. Thermodynamic equilibrium is an asymptote, not a state. Particularly in this case, with transport limiting, I wouldn’t expect equilibrium to be very meaningful. All it does is crank up the transport driving force, and speed up the transport processes by some amount.

      • Peter317, I would put it this way. The whole adjustment by 1 C is supposed to lead to a greater radiative flux by 3.7 W/m2 at the top of the troposphere to cancel the negative effect there from doubling CO2, so finally everything is back in balance at the top. To maintain this balance, the whole column is 1 C warmer and has to stay that way. Any cooling would lose the balance again.

      • ChE, in a transient state one view is that it is possible RH will initially decrease because the ocean warms less quickly than the land. This might lead to droughts and less cloud cover, and is not necessarily something to hope for.

      • The point is, that equilibrium is NEVER reached. It’s always in a continuous steady state of disequilibrium, with transport processes keeping equilibrium at bay. If that has other ramifications, it is what it is, but equilibrium calculations are a limiting case, not an accurate model.

      • pekka,

        Thanks for the reply. I had many years of using steam tables. But the situations are here different with 1 degC temperature rise. Those saturation pressures will never achieve in the atmospheres unless it is about to rain locally and I reckon even at sea surface, seldom achieve 100% RH or saturation water vapor pressure un less it is about to rain soon locally.

        Even consider saturation pressures at 19degC and 20 degC atmospheric temperatures corresponding to 0.022 and 0.0234 bar saturation pressures respectively water vapor pressure, 1 degC rise corresponding only about 5% water vapor increase and not 7%.

      • Pekka,

        What you write is true for the total gross flux from the surface, but there is also down-welling radiation that increases with increasing temperature of the atmosphere. Thus the net flux does not change as much, not even nearly as much.

        The down-welling radiation cannot be more than 1/2 of the proportion of the up-welling radiation which happens to be within the CO2 absorption band.
        But, regardless of whether or not this is enough to re-balance things, you still need to get the temperature increase before you can get the increase in down-welling radiation.

      • Sam,
        Mostly I skip answering to you, because you never accept the facts as they are, but always finds new excuses for continuing to disbelieve, whatever you are told.

      • Jim, It’s all very well saying that you have to increase the temperature by 1C to re-balance the radiative flux at the TOA, but, unless you have a mechanism to provide the extra energy required at the surface, it just ain’t going to happen. Either the imbalance will persist, or something else will break – but you can’t just ‘magick’ the extra energy.

      • Jim D,

        “SamNC, the C-C formula is de/e=dT*Lv/(Rv*T*T)
        where e is that saturated vapor pressure, so de/e is its relative increase, dT is the temperature increase, Lv is the latent heat of vaporization=2.5e6 J/kg, Rv is the gas constant for vapor=460 J/kg/K, and T is the temperature in K.”

        Where did you get those constants, Lv and Rv for water vapor?

        Lv is not a constant for water vapor. It varies depends on T in Celcius as shown above valid between -40 to +40 C. And for easy reference:
        Lwater(T) = − 0.0000614342T3 + 0.00158927T2 − 2.36418T + 2500.79

      • This is not a particularly fruitful line of argument. In climate models the integral relation of e and T is typically given by 10th order polynomials that account for the T dependence of Lv and other effects, see for example
        http://rams.atmos.colostate.edu/cotton/vita/71.pdf

        The mechanism for the 7%/K water vapor increase is described in my feedback chapter, see section 13.3 and esp following eq (13.23)
        http://curry.eas.gatech.edu/climate/pdf/Ch13_GalleyC.pdf

        See Held and Soden for what climate models produce
        http://www.coaps.fsu.edu/pub/williams/GRAD/Global_CLimate_LandUse/Still_Grindin_2009/Held_2006.pdf

        As for “real world” observations of water vapor increase, see Wentz et al.
        http://www.remss.com/papers/wentz_science_2007_paper+som.pdf

        There may be some more recent papers on this that I dont currently have time to track down.

      • pekka,

        “Mostly I skip answering to you, because you never accept the facts as they are, but always finds new excuses for continuing to disbelieve, whatever you are told.”

        1. I don’t mind you skip. If they are facts show me.
        2. If you think I am wrong, prove me wrong instead of expecting me to be gullible to believe “whatever you are told.”
        I have shown you that 7% is exaggerated in my reply with your above link to determine saturation water vapor pressure, proof me wrong that 7% is not exaggerated.

      • Peter, the flow of energy from the sun to earth and back out again supplies this continuously. All CO2 does is block some outflow leading to a back-up of energy, seen as a temperature increase. There are analogies to faucets, sink water levels, and plug-holes to visualize this.

      • SamNC,
        Lv is close enough to constant since it varies by 1 part in a 1000 per degree. Rv is the universal gas constant (8310) divided by the molecular weight of H2O (18). I am sure you can find something about Clausius-Clapeyron on the Web to explain all this. The C-C relation depends on temperature squared, so for warmer temperatures it might be only 6%.

      • Jim, as you’ve now attempted to fob me off with some woolly oversimplification, I’ll just assume that you have no interest in discussing this matter any further.

      • Peter, your questions are pointing to a misunderstanding of the energy flow. I tried to correct it. The energy flow is a hundred times larger than the imbalance, and thus can easily maintain a new equilibrium state.

      • Now you’re becoming insulting. Just because you may have misread something I wrote doesn’t mean that I don’t understand energy flows.

      • JC wrote:

        As for “real world” observations of water vapor increase, see Wentz et al.

        This is a link to the paper.
        How Much Will Precipitation Increase With Global Warming?

      • oops here is the correct link, will try to find original
        http://www.remss.com/papers/wentz_science_2007_paper+som.pdf

      • Peter, I was struggling with your concept of ‘extra energy’. Try this explanation. Forget the atmosphere, and have a grey body (albedo 0.3) heated by the sun. Now increase the net incoming energy by 3.7 W/m2. The grey body warms slowly by a degree to come back into equilibrium and only then stops warming.

      • OK, one last try, using your bathtub analogy:

        You have 3.7gal/min more flowing into the bath. In order to get 3.7gal/min more flowing through the plughole (labelled ‘radiation’) you need the extra pressure due to the depth of water increasing by 1ft.
        But there are also another two plugholes, one labelled ‘convection’ and the other labelled ‘evaporation’ . Because you now have extra outflow through these two, the water level never gets to the 1ft higher level.

      • Peter, the problem with that is that the water level does not depend on convection and evaporation because those are internal to the water and they cannot remove energy from it. The water level is the temperature of the earth system including atmosphere, water and land. Convection and evaporation are only distributing energy within the system.

      • Dr Curry,

        Thanks for the links to bring back the right track.
        I took a glimpse of the links. Here is my impression:

        http://rams.atmos.colostate.edu/cotton/vita/71.pdf
        Very nice about the steam table, real experts.

        http://curry.eas.gatech.edu/climate/pdf/Ch13_GalleyC.pdf
        Your eq (13.23) did not provide a description of what the terms are and did not have an example and then concluded 7.7%/K. Will you explain.

        http://www.coaps.fsu.edu/pub/williams/GRAD/Global_CLimate_LandUse/Still_Grindin_2009/Held_2006.pdf
        So doubling of CO2 is anywhere causing extra evaporation from 1%/K to 9.1%/K. As I said I did not go into detail reading of these links, my impression these models have spurious evaporation results as wellas those fluxes numbers from 0.6W/m2 to 3.7W/m2 by doubling CO2.
        ttp://www.remss.com/papers/wentz_science_2007_paper+som.pdf
        Is it real world? No substantiation of measured data in the link.

      • Peter,
        What made you conclude that the downwelling IR radiation from the atmosphere cannot be more than half of the emitted, when it is well known from direct measurements that its usually much more (this is confirmed every time a pyrgeometer is used), and estimated by calculations to be on the average more than 80%.

        Why should one thing happen before the other, when the natural way is that they proceed simultaneously. Of course the change is initiated by the external forcing, but the feedback reaction follows as soon as even a small change has occurred.

      • Jim D, Do you not understand that I’m talking about what happens at the surface, even though I’ve made that quite clear from the beginning?
        Or are you deliberately obfuscating things by shifting the goalposts?
        Either way, I’ve said my piece so I’m not interested in continuing this discussion.

      • Pekka:

        What made you conclude that the downwelling IR radiation from the atmosphere cannot be more than half of the emitted

        Sorry, my bad. I was getting confused with what happens at a molecular level – a photon has an equal probability of being emitted downwards as upwards. It was a long day and I was tired. ;-)

        However, you still can’t say that a large proportion of the 3.7W emitted by the surface is re-radiated back to the surface, because of the simple fact that the original 3.7W extra received by the surface is already back-radiation. It’s the amount of extra back-radiation down-welling from the atmosphere to the surface as a result of the doubled CO2 concentration
        That extra 3.7W must come from down-welling radiation, as there’s nowhere else it can come from.
        So to say that a large proportion of the extra 3.7W emitted by a 1C warmer surface is then returned to the surface by way of back-radiation is double-accounting. The back-radiation proportion of the energy has already been accounted for by the time the first LW photon strikes the surface.
        The actual proportion of the extra 3.7W from a 1C warmer surface which would then be re-radiated to the surface would be in the order of 1% (3.7W/333W, the 333W coming from K&T)
        So with the extra losses from convection and evaporation factored in, the net extra flow to the surface due to doubled CO2 would not be nearly enough to increase the surface temp by 1C.

        As to things happening simultaneously, a molecule has to receive a photon before it becomes excited, and it has to become excited before it can emit a photon.
        I only say that x must happen before y to counter the arguments which start out with the assumption that y has already happened and that x must then somehow adjust itself in order to result in y.

      • Peter,
        Your latest comment tells clearly, how the overall changes build up of many components. Starting from one leads to changes in another, etc. The physicists way of putting the whole in order is to write down all equations that connect the significant variables. That is precisely the way it’s done in textbook presentation of these matters.

        Trying to make shortcuts leads to mistakes at some point. It makes one to think, that standard presentation would be in error, but the error is not there but in some factor forgotten in the attempt of making the shortcut.

        It’s not possible to describe things fully in a blog discussion. Even the basics involve too many details for that. Others may try to answer specific questions. Sometimes these answers help in getting the issue clarified, sometimes not. Very often it’s not possible to really see, where the problem is, and then the answers are usually beside the point.

        The only real solution is to start from the beginning using some textbook presentation or similar net material. If some point is difficult to understand or accept, then it’s at least possible to tell, where the difficulty arises, and getting help may be easier.

        Starting from statements telling that the standard theory must be wrong, because of “A”, is often fruitless, because others cannot really see why the statement “A” is taken so seriously. Therefore their answers fail to clarify the point.

    • TimTheToolMan

      Jim D Writes : “Doubling CO2 is equivalent to raising the solar input by 1% and sustaining it.”

      No its not, they’re fundamentally different. Raising solar input is increasing the amount of energy that reaches the earth whereas doubling CO2 acts to slow the amount of energy that leaves. They’re quite different with quite different implications on feedbacks.

      • andrew adams

        I don’t really see why. Either way it basically comes down to changing the balance between energy in and energy out – what difference does it make whether that change is due to an increase in one or a decrease in the other?

      • Just spurious data, nothing serious, i.e. pseudoscience.

      • andrew adams

        What data is spurious? Incoming solar radiation, outgoing or downwards LWR? These things are all measured and are pretty uncontroversial.

      • a a ,

        I presume that you did not read K&T’s 1997 Global Annual Mean Radiation Budget. Take a look at 390W/m2, 324W/m2, 35W/m2 … etc. Come back if you can account for any of these data.

      • aa,

        “downwards LWR” measured? uncontroversial? Show me!

        Don’t confuse with modeling results and measured.

      • aa,

        That science of doom did not show the measured, mere heresay, no link to a real measurement, no details about the methodology, instrument used in the case of Ontario. Don’t confused with measuring sun radiation with back radiation (or DLWR)

      • aa,

        After visiting science of doom, I have very much doubt in those DLR measurements methodology. The pyrgeometer is supposed to measure radiation at night to avoid error from the sun radiation. It was wrong to assume the sun radiation reaching earth only involved SW,i.e. 4um are also significant (could be the major contributor), but science of doom and other people involved in the DLR measurements conveniently ignored the effect the Sun’s LW radiation.

        Only when those DLR measurement papers are available for public scrutiny to estimate/assess to their error of measurements involved, so that we can have a better idea if back radiation is really there and how much.

      • Looks like the pyrgeometer was sensing the immediate air temperature rather than the downward long wave radiation according to its construction.

      • Correction – the pyrgeometer measured the effect of air temperature and the Sun’s LW radiation.

      • See World Radiation Center on the use of the pyrgeometer and its calibration:

        http://www.pmodwrc.ch/pmod.php?topic=irc

      • over time, the energy leaving the earth must always be equal to the energy coming in. thus, adding CO2 to the atmosphere cannot change the energy leaving the planet. increasing the sun’s energy increases the energy coming in, and thus the energy coming out, so the two processes are fundamentally different.

      • This is the fundamental concept. Adding CO2 restricts the IR energy going out until the earth warms enough to restore the balance.

      • Jim D,

        ” Adding CO2 restricts the IR energy going out until the earth warms enough to restore the balance.”

        Again, I did not see your statement with substantiations. Without substantiation is spurious. I am what trade you are in. Statement without substantiation is not science. You should start with 0.039% CO2 content physical properties of CO2, i.e. specific capacity of CO2, then specific capacities of 79% N2, 20% O2, H2O … to quantify the energy/radiation magnitudes of each gas involved. If cannot give a good account of these gases involved, its not science, just spurious data.

      • Sam NC, you are behaving as though everything I say is new. It is mainstream science.

      • Jim D,

        You are correct they are new or weird to me in accordance with real science and physics. Mainstream pseudoscience non-conformance to science and physics can do whatever they want with consensus.

        You are avoiding all direct questions and your answers so far no substantiation. Your answers are easily misformed gullibles. I am prepared to accept warmists claims if they can substantiate any numbers.

      • There aren’t going to be any well chosen sentences from me that teach you physics. You have to teach yourself. I just give you the bottom line. You have to look for the rest yourself, if you don’t believe me.

      • “Adding CO2 restricts the IR energy going out until the earth warms enough to restore the balance.”

        Yet this is not what we observe when looking at mars and venus. Their high CO2 atmspheres have not increased their temperatures after correcting for pressure and distance from the sun.

        This suggests that the atmosphere of a planet is not as simple as the standard IR model. It also suggests that climate science could learn a lot more about climate by studying real planets rather than playing with imaginary ones on their computers.

      • Venus has an albedo of 0.7. We don’t expect it to be so warm when you account for that. This is a common mistake going around the blogs. I wish it would be corrected.

      • And we certainly don’t expect it to be so warm when you account for the fact that little or no solar radiation reaches the surface.

      • It wouldn’t make any difference if you had steady-state conditions. But they’re not, so it does.

      • This begs a question.

        If you double the amount of Iron in an Iron rod, you have essentially doubled the ‘Radiative Resistance’ of that object while doubling it’s ability to conduct.

        Now if you increase the Radiative Resistance of an atmosphere, and do nothing to restrict the Convection, have you in fact reduced the overall cooling rate?

      • This is how forcing is defined. It can be changes in ice-cover albedo, solar input, GHG concentrations, volcanoes, aerosols. The forcing number is what matters. The IPCC report starts with the idea of forcing because that is so fundamental to understand first. It weighs these factors against each other. As I said, if it wasn’t for the degree of new forcing from CO2, it would not matter, and would just contribute to climate noise.

    • How now round cow?

  16. None of that relieves individuals or groups from exercising QC on the elements they have direct involvement with and knowledge of.

    You can’t build a palace, or even a two-story house, out of rotted wood.

  17. Instead, there will be a large group of people who collectively claim to understand all the separate pieces that go into the discovery, and how those pieces fit together.

    JC: This would explain the IPCC “consensus” and the large number of people that support it.

    Wouldn’t it also explain the certainty of the “climate scientists are frauds” arguments? How about the certainty that “climate scientists are statists pursuing a one-wold government that will dictate who has access to energy” arguments?

    Shall I go on?

    How is it, Judith, that you read comment after comment, day after day at this blog, and miss the group of people that collectively claim to understand all the separate pieces that fit together to explain why the Earth isn’t experiencing anomalous warming; or that if it is experiencing anomalous warming it isn’t anthropogenic in nature; or that even if it is experiencing anomalous warming that is anthropogenic, we can’t do anything to prevent it; or that even if it is experiencing anthropogenic warming that we could do something about, the costs of preventing the warming would outweigh the benefits?

    • We don’t claim to know how it all fits together.
      We simply point out that what is being sold by the AGW promotion community does not fit together.
      That you confuse the two speaks volumes.
      1-Try this on for not fitting:50,000,000 climate refugees by last year.
      2- Himalayan glaciers not melting by 2035.
      3- Coral reefs destruction > 10% by 2010.
      4- Manhattan experiencing changes in vegetation and being flooded by 2010.
      5- More frequent and stronger storms by now.
      6- ice free Arctic/grave outcomes from reduced ice in Arctic
      to name a few failed AGW predictions.
      Not one actually happened, of course. Skeptics do not know why they did not happen.
      Skeptics point out that they in fact are wrong.
      True believers like yourself want to blame the skeptics for pointing this out, or pretend the predictions were not made, or twist things around and claim they were not made in peer reviewed journals so don’t really count.
      Fred pretends that pointing this out is a creationist plot to make things controversial and hide the truth by distraction.

      • Sorry, hunter, I guess I was mistaken.

        I thought that you claim with certainty that significant anthropogenic global warming isn’t happening.

        Oh, and btw, hunter, just because in your imagination I am a “true believer,” doesn’t make it so. Interesting, though, that in that very post you claimed certainty about something without having sufficient data to draw conclusions.

      • Joshua,
        Nice filibuster.
        I point out that to date there is no evidence of a global climate crisis caused by CO2.
        I make a list of failed claims and predictions of that global crisis and you, understandably, try to shift the point to something more your style.
        You twist other people’s words well enough to pass for a true believer.
        If you are skeptical, you hide it well.
        Now do you want to make a sidebar to Dr. Curry about how I compare AGW believers to eugenics believers, and ask her if she is actually reading my wicked comments?

      • hunter,

        You have indicated numerous times that I am a “true believer,” or some such nonsense. I just figured since you make such definitive conclusions you must some reason.

        I’m very curious to find out what I believe that I never realized I believe.

        Do tell – what are my opinions that make me a “true believer?”

        This should be good.

      • Joshua,
        Faith without action is dead, to paraphrase someone.
        Your actions speak of your faith.
        Do you ever deconstruct an AGW claim?
        Do you defend an AGW failed prediction?
        No. You ignore it and try to deflect the conversation to something else, like how terrible it is for the similarities between eugenics and AGW to be pointed out.
        Or how I dare to call you a believer.
        So you have two of the major aspects of the true believer:
        Unwillingness to deal with direct issues that run counter to the faith, and cowardice to stick to issues.

      • hunter.

        You’ve posted more than once on the topic, but you have still failed to explain to me which of my opinions make me a true believer.

        Shall I assume that your failure to do so is an admission that your claims (made without evidence) are wrong?

        And hunter, you seem to be under the mistaken impression that I am offended by your unfounded characterizations of me.

        Quite to the contrary. Such characterizations only help to make my point. Your behavior is a prime example of “tribalism” among “skeptics/deniers.”

      • Joshua,
        You are of course free to assume anything you wish.
        But, like assuming that the world is facing a climate crisis because CO2 is a ghg, that would be an assumption without evidence.
        But now the mundane world calls, so we will continue enjoying your dodge and weave later.
        Cheers,

      • You are of course free to assume anything you wish. But, …,that would be an assumption without evidence.

        Anytime you feel like it, hunter, I’d be happy to read about which of my opinions make me a “true believer.”

      • True climate believer,
        Hidden knowledge persuader.
        ==================

      • Joshua,

        A bit off-topic, but I see you persist in using the loaded term “denier” despite many courteous requests to refrain from that scurrilous term. Some have even suggested a that retaliatory term should be devised. My notion is that instead of a single term of contempt for the likes of you, the better reponse would be to tailor a specific term for the creep in question.

        For example, there’s a real jerk-off named greenfyre (who has an obscure blog no one reads except when linked from one of the respected, prestigous blogs like this one). I’ve devised the following term for greenfyre:

        “Princess Climate-Polyp von Thermorrhoid”

        In referring to greenfyre by the above title, I hope the reader, through subliminal associations, will come to view greenfyre as a girlie-man control-freak and a royal pain-in-the-butt.

        Right now, I’m working on some sort of little name for you, Joshua–any suggestions you might want to offer?

      • Mike –

        Have at it. I really don’t take any offense to whatever term you use to refer to me. hunter and some of his buds have referred to me a “warmist,” or some such nonsense, ever since I first showed up in these here parts, so I doubt that whatever term you come up with, it would be particularly novel, or uncharacteristic in comparison to many other folks here that generally share your perspectives.

        But I will point out that I use the term “denier/skeptic.” As far as I’m concerned, anyone would be perfectly entitled to identify with whichever side of that slash line they prefer.

        My working assumption is that there are some folks out there who do fit the term of “denier.” In other words, people who, for political, or ideological, or religious reasons, disregard any scientific evidence that would contradict their predetermined opinion re: climate change. I also assume that there are many out there that would be better described as a skeptic, in that they do care about the science and do not feel that as yet we have sufficient evidence to back up theories that anomalous global warming is occurring, and that it is probably anthropogenic in nature.

        Unless I sat down and talked with any particular individual, I would have no way of knowing whether the term “denier” or “skeptic” would better describe their viewpoint. Unlike hunter, I do not make assumptions about someone when I don’t have sufficient evidence (well, at least I try not to do so).

        If the term “skeptic” is more appropriate for you than “denier,” please feel free to identify with that side of the slash line when you read one of my posts. If I have failed to include the “skeptic” side of the slash line in one of my posts, I apologize for having done so.

        Anyway, I look forward to your attempts to creatively find ways to insult me. I’m sure that they will be equally as amusing as have the insults of hunter et. al.

      • Joshua,

        You know, Josh, it’s a little hard to get an angle on you. With greenfyre, it’s easy. Greenfyre is a trashy low-life. A joke. He knows it and I know it. So I use a language with greenfyre that is appropriate to a clownish loser like him (though he sure does have a knack with them there “purty pictures” of his).

        But you, Joshua, your shtick (a good one, by the way) is of a different stripe. Good ol’ Josh–the voice of scholarship and moderation and sweet reason. I mean, like, reading through your comments, ol’ buddy, one can imagine you have a whole wall-full of humdinger degrees and awards. Bow-tie and tweeds type of guy. A man of science worthy of respect and deference–that sort of thing. Like I say it’s a good act. But the “denier” business is the give-away.

        You’ve been told by those most abused by the “denier” term that they find the term to be offensive and you’ve received courteous requests not to use the term. And the reason given is that the term is suggestive of “Holocaust denier.” And, as we both very well know, such a term with such associations crosses a bright line in a way “warmist”, for example, does not. Again, you know that, right Josh? –glad you’re not playing dumb on me.

        Now, in the face of such an appeal and explanation, no individual of good-will, common courtesy, and just plain decency would persist in using the word “denier.” On the other hand, a certain type of agit-prop slicko would see it differently. You know, like some pretend nice-guy who just can’t bring himself to jettison the best and only successful propaganda tool at his disposal. And why? I mean what is it with you guys? What is it about your CAGW pseudo-religion, Josh, that turns individuals of intelligence and with a naturally moderate disposition into such nasty pieces of work?

      • Bow-tie and tweeds type of guy.

        Mike – that’s hilarious. You have no idea just how completely wrong you are on that front.

        I don’t use the term “denier” to equate anyone to a holocaust denier. I find that sort of hyperbole to be counterproductive – just as I do when someone analogizes climate scientists or environmentalists to Eugenicists. I defined how I use the term “denier” (some people use the term denialist if that would make a difference), and I make it clear to offer it as distinguished from a “skeptic.” The point being, I don’t use the term “denier” to refer to skeptics, and I don’t use the term “skeptic” to refer to deniers.

        I have no problem with the term “warmist,” per se, because I know that it doesn’t apply to me and I pay little mind to politically correct concerns. I assume that the term “warmist” isn’t all that inaccurate for some folks. My problem is when people ignorantly use that term inaccurately, because they lack sufficient knowledge to know whether their use of that intended-to-be-derogatory term is even remotely appropriate.

        Mike – if you insist that I’m referring to you when I use the term denier, even though I explained how I define the term, and even though my definition has no reference, whatsoever, to holocaust denial, that’s on you.

      • Joshua,

        You know, Josh, you’ve got an unflappable quality that I can’t help but grudgingly admire. My compliments.

        Also, I need to consider this issue more from your point of view. The S. S. Klimate-Klaptrap has taken the final plunge. The skipper and mates have motored off over the horizon in their life-boat yacht (Cap’n Al was the first to abandon ship), and all you have to cling to and keep you afloat is your “denier” bit of flotsam. So I appreciate we’re talkin’ survival here.

        So the “bow-tie” business was a miss? I knew it was a long-shot, but I didn’t think it my place to speculate too wildly about the contents of a fellow commentator’s closet. I do like the way you handled it though–I mean, leaving your sartorial preferences up to our imagination and all that.

      • Dance around the truth,
        Words mean what we say they mean.
        Who pays the piper?
        ===========

      • kim is omniscient
        I believe as he describes
        well, in universe Kim

      • I think I’ve never heard so loud
        The quiet message in a cloud.
        =============

      • Government Agencies paid them with hard earned via taxes.

  18. While the discussion on complexity of knowledge will be fun, I believe the disciplines of science and economics are fundamentally different.

    Economics theories are relatively trendy and economics (as the “dismal science”) is really a description of human behavior in the marketplace. This is why there is hidden knowledge, since every scenario in which humans and human organizations make decisions is complex.

    Fundamental laws of science (like mathematics) on the other hand have been demonstrated to be fairly robust and enabled large bodies of human endeavour which would have been otherwise incomprehensible a century or two ago (flight, space travel, computers, etc). While the nitty gritty of the science underpinning modern microelectronics and telecommunications need not be understood by everyone, that they can use the products which arise is evidence of the soundness of the underlying science.

    If a paleoclimatologist thinks there is a 50% chance the earth is warming as a result of human activity, I believe it. If a climate scientist claims it is almost certain that the earth is warming because of human activity, and that rising levels of CO2 are the cause, and that we need to tax carbon to fix it so we can all put in more nuclear power stations over the globe , I don’t believe it. That every one of those assertions could be wrong makes the chance that all of them are right rather low.

    • You say that the probability of the last three being true is rather low. But the first two you offer – “That the earth is warming due to human activity, and that rising levels of CO2 are the cause” are – if you substitute ‘GHGs’ for CO2 (the relative proportions are fairly well known) is essentially the same as the proposition you were willing to accept had a 50% probability is true. But on a 50% chance of that being true, some effort to reduce emissions seems warranted, and an emissions tax is widely acknowledged by economists to be the most efficient way to reduce GHGs. So now, aren’t we simply arguing about the right size of tax, given the uncertainty?

      • There is a logic jump from facts/hypothesis to tax.

      • There is a logic jump at each step where there is dispute over the answer. Simple probability: 50% of 50% of 50% of 50% = 6.25%

      • Well I would say less than 1% of 1% of 1% of 1% approaching zero!

  19. “No single person understands all of this, except in broad detail.”

    That is the definition of the Tax Act. No individual knows the Act, yet under Law ignorance is no defense.

    • And “broad” and “detail” are incompatible words. Like “thick skinniness” or “rapid slowness” or …

      ;)

  20. Hi Dr. Curry and other letter writers,
    I’ve been lurking here for a while and I’d like to join in the conversation. I found the article to which Dr. Currie referred interesting. I wonder how many people know what it is like to study the climate on a day to day basis. There are so many scientists and students from many countries working on understanding climate. Everyone learns the basic physics. But when one gets to the real research, one is confronted with two complex tasks: analyzing the model output and analyzing the measurements. Both tasks involve averaging and assumptions. Some study their own back yard, some look at the globe. The “hidden knowledge” is not necessarily hidden, it is just spread out over many scientists.

    The hope (or delusion) of SOME of the scientists is to collect this dispersed information and call it AGW. I have lost confidence in them because they have brought politics into the science.

    Many scientists, however, just enjoy doing the research because the climate is a fascinating puzzle. These I trust.

    • “Everyone learns the basic physics.”

      As an independent physicist, I am here to tell you that you, and your entire generation, have been miseducated. Your textbooks on radiative transfer theory are worthless, and worse. There is definitive evidence that inicreasing CO2 does not increase the global temperature (I have provided that evidence myself; you’re reading the words of the guy who KNOWS this crucial fact, and will never allow it to be forgotten, so long as I have the wherewithal to proclaim it). The science is not too complex; climate scientists have been drawn into the pit of false theory, and concerted and ever-growing cover-ups of their scientific incompetence, and are too shocked or lacking in basic character (honesty) to face that fact. First, stop digging that pit deeper. Shut down the political “climate policy” machinations, which are not pointed toward the truth. And shut down the excuses, that the politically correct (like Judith Curry) keep throwing up. You can’t trust any climate scientist who is not screaming to high heaven that the “consensus” is a monumental fraud, and that all of climate science is but a treacherous mire of unquestioned false dogma — false “basic physics” — pulling down all of your efforts to find and understand that which is your only real goal: The truth. To call the discussions that are going on “reasonable” is simply pitiful. You (all scientists) are up against irrational avoidance behavior, and if you’re not focusing on that fact, it is because you haven’t looked into the critical evidence against the “consensus”, and should not say anything until you do, or you are avoiding the truth yourself. I don’t base my statments on theory, only the critical, definitive evidence. That is what science direly needs, and what too many scientists are not providing. They are incompetent.

    • Hi Rose – welcome to Climate etc. :)

  21. Eksperimentalfysiker

    The following quote from Erwin Schrödinger seems appropriate:

    “A scientist is supposed to have a complete and thorough knowledge, at first hand, of some subjects, and therefore, is usually expected not to write on any topic of which he is not a master. This is regarded as a matter of noblesse oblige. For the present purpose I beg to renounce the noblesse, if any, and to be freed of the ensuing obligation. My excuse is as follows. We have inherited from our forefathers the keen longing for unified all-embracing knowledge. The very name given to the highest institutions of learning reminds us that from antiquity and throughout many centuries the universal aspect has been the only one to be given full credit. But the spread, both in width and depth of the multifarious branches of knowledge during the last hundred odd years has confronted us with a queer dilemma. We feel clearly that we are only now beginning to acquire reliable material for welding together the sum-total of all that is known into a whole; but, on the other hand, it has become next to impossible for a single mind fully to command more than a small specialized portion of it. I can see no other escape from this dilemma (lest our true aim be lost forever) than that some of us should venture to embark on a synthesis of facts and theories, albeit with second-hand and incomplete knowledge of some of them, and at the risk of making fools of ourselves. So much for my apology.”

    Preface to ‘What is Life?’ by Erwin Schrödinger
    http://whatislife.stanford.edu/LoCo_files/What-is-Life.pdf

  22. “I thought that you claim with certainty that significant anthropogenic global warming isn’t happening.”

    The earth has been warming since the Little Ice Age. The current rate of warming in the past 100 years is no different than what occurred 300 years ago.

    So the question is, when did the warming from the Little Ice Age stop? How can we be certain that it did? How can we know with any certainty that what we are seeing today isn’t part of the warming that started 300 years ago?

    Have a look at the oldest thermometer record available (link below). The rate of increase today is if anything less than in 1700.

    http://en.wikipedia.org/wiki/File:CET_Full_Temperature_Yearly.png

  23. The best “Hayekian” essay on science is probably Polanyi’s “Republic of Science.” He gives a compelling account of how a decentralized community of independent researchers can make all kinds of judgment calls and deal with complex, ambiguous data while being guided, as if by an invisible hand, toward the growth knowledge. Unfortunately, a key assumption he makes–that each researcher is primarily concerned with his own discoveries and reputation–breaks down in a world where funding of research is conditional on one’s findings and/or one’s findings will influence policies that affect society. In that circumstance, researchers’ incentives no longer line up perfectly with using their own best judgment about the truth.

  24. “Might we one day find ourselves in a situation like in a free market where systematic misunderstandings can infect our collective conclusions?”

    That day arrived some time ago.

    And the use of the term “free market” is rather quaint, but useful for this discussion.

    Perhaps relevant, the “free market” consensus is often wrong, sometimes spectacularly wrong.

  25. I think this article makes a bad assumption off the bat. It seems to assume or at least imply that all science barring climate science is quite simple and easily understood by one person.

    Climate science however is far too complicated for an individual and therefore we must rely on the IPCC and the experts for their judgement. This is clearly incorrect, as an early post of mine highlights.

    I think there’s an issue here that has been touched upon perhaps tangentially by the OP, but is nevertheless very important.

    The problems that climate science is currently enduring can all be traced back to procedural problems. The accuracy of the results, the error limits, the reproducibility and the validation (of models, equipment, data etc) are all ‘coming back to bite’ as the theory is laid bare more and more to the world examine the claims and the supportive data/methods.

    The (correct) call for an engineering style report is actually also asking for something more basic; proper scientific procedure.

    Now by this I am (of course) referring to the industry standard procedure for scientific work. Much of the academic work I’ve encountered (in all fields) is, to be kind, weak on methodology and basic procedural care. I know of course that there are exceptions and that to tar all academia as one thing is no worse than tarring all skeptics as another, but I think (and have stated frequently) that Academia can and MUST learn from (scientific) industry to survive.

    Were the ‘proper’ procedures to be followed in academia and specifically climate science as a field, we would not be having half of the discussions we’re having now.
    If you came to my lab and checked a labbook for a specific project I could (within minutes) give you all the supportive raw data, show you the maintenance and calibration logs for ALL the equipment I used, how they were validated, I could demonstrate the methodology and the justifications for said methodology and you’d see every single calculation and it’s reasons in full detail (with the data pre and post for independent verification). More importantly I highlight all the problems and the possible tests that could DISPROVE or ‘damage’ my work. Further, all the data, the files, the write-ups and corresponding emails (natch) are all archived ready for a full audit at ANY time. Finally all my work is independently checked by our QA team.

    This does two things- 1) it protects ‘us’ from any accusations of wrong doing and 2) ensures exceptionally high-quality data and science.

    Now I am by no means trying to suggest that academia adopts all these procedures, but some- especially the data integrity stuff, would help endlessly and it is very simple to implement.

    So to address the accuracy and the validation points in a very roundabout way- rather than trying to ensure that the data at the end is ‘sound’ we should be concentrating on making sure that the methodology and procedures followed are sound- that way you don’t have to worry about the accuracy or the validation as the controls are already built in.

    • I wholeheartedly agree with you.

      Most of this should have been taught and internalised by the end of a B.Sc. course. It should have been learned by every single student who aims to go on for a postgrad degree.
      Sadly, even twenty years ago this was no longer the case where I worked. Supervising a final-year student, as post-grad, my nagging about him not grasping the difference between 10*6 and 10*7 fell on deaf ears – and the prof responsible more or less directly implied to overlook this because, hey, it would mess up the note for this piece of work and would be bad for the guy …
      So if academe even then accepted that a figure somewhere in the ballpark was ok even if not precise – one shudders to think about the competence of those leaving our universities now.

  26. “Might we one day find ourselves in a situation like in a free market where systematic misunderstandings can infect our collective conclusions? How can we be sure the results of large-scale collaborations or computing projects are reliable? Are there results from this kind of science that are already widely believed, maybe even influencing public policy, but are, in fact, wrong?”

    I wonder if the author of the article really grasped what Hayek wrote? He cites an article that discusses at length the superiority of a free market system over central planning when it comes to utilizing knowledge, and apparently only takes away from it the concept of “hidden knowledge.” (Note that Hayek’s article is titled The Use of Knowledge in Society, not The Nature of Knowledge.)

    Hayek’s point was that a free market economy is more efficient than a planned economy both because it provides a mechanism for processing more knowledge from more sources, and because it is more efficient at dealing with new information, and correcting old, incorrect information.

    In the cited article, Hayek wrote: “It is possible today to minimize the importance of the knowledge of the particular circumstances of time and place, this is closely connected with the smaller importance which is now attached to change as such.” His point was that central planners seriously over estimate not just their own knowledge, but their ability to construct their plans to adequately cope with rapidly changing new facts. The plan itself hinders the communication of, and therefore the use of, newly discovered or changed knowledge.

    The “systematic misunderstandings” Leinen fears can and do arise under either system. The question is which system can recover from such mistakes more efficiently. By providing a superior means of communicating knowledge from all sources, a market system is far superior in that regard to anything that a centrally planned system can hope to accomplish.

    Imagine if Michael Mann and James Hansen worked in a market oriented system, rather than being funded by governmental central planners. Would their serial errors be met with such callous disregard? Would their employers be so impervious to the knowledge provided by the skeptics, contrarians and McIntyres of the world? Better yet, imagine a centrally planned global scheme to deal with CAGW. Just how responsive would such a plan be to changes, new information, or the discovery of errors?

    Hayek’s point was not to explain the nature of hidden knowledge, but to explain why the existence of such knowledge in an extremely complex system makes central planning a poor choice. A free market is both better suited to arrive at correct decisions, and more capable of correcting mistakes. The more complex the system, the more important it is to allow a free market mechanism to process knowledge held by everyone.

    Implicit in Leinen’s fear is his apparent belief that not only will large scale complex research lead to results that no one person will be able to know alone. But also that large scale policy decisions will be made based on those decisions. If you really understand what Hayek was saying, the safest way to deal with such wide ranging, complex hidden knowledge is to take the decisions to be made from it out of the hands of central planners. Proper understanding of that could have alleviated some of Leinen’s concern.

    (I must say that is refreshing to see Hayek discussed at all in a forum like this though. I managed to complete four years of liberal arts education in the 70s in political science, history, economics and philosophy, without hearing the man’s name mentioned once. Ah the wonders of progressive education.)

  27. By contrast, until quite recently the complete justification for even the most complex scientific facts could be understood by a single person. ………
    Science is no longer so simple; many important scientific facts now have justifications that are beyond the comprehension of a single person.

    ?
    Core of scientific progress:
    Analysis of data.
    Synthesis of ideas.
    Science is built on reasoning of an individual !

  28. David Bailey

    Many subjects are too complex for one individual to understand – think of the desktop computer – but normally knowledge is carefully compartmentalised with interfaces between the various parts. Thus I can program my computer without regard to the electronic structure of the CPU chip, or even the functioning of the microcode.

    I sense that part of the problem with climate science is that these knowledge compartments have broken down in a very dysfunctional way. Thus some climate scientists become amateur botanists (tree rings), amateur statisticians, amateur meteorologists, amateur computer programmers etc.

    In the end they know they are bluffing most of the time, and that makes them become more and more insecure and secretive.

  29. “But such collaborations will be no good if we can’t assess the reliability of the results. And it would disastrous if erroneous results were to have a major impact on public policy. We’re in for a turbulent and interesting period as scientists think through what’s needed to arrive at reliable scientific conclusions in the age of big collaborations.”

    We all know CAGWers trying to prevent FOI to access their data and calculations to prove they are inadequate, immature, subjective and possibly wrong. These data were supposed to belong to the public as these were on public tax money. Just this fact is enough to query they are ethical.

    A typical example of hiding calculations was the K&T’s 1997 Global Annual Mean Radiation Budget hiding the calculations to get to all those numbers such as 390W/m2 Earth Surface Long Wave Radiation, 324W/m2 GHG Back Radiation, 35W/m2 CO2 radiation contribution. If they could show the calculations, these data would be immediately busted/debunked, sad AMS peer review process spreading misinformation for almost 2 decades.

  30. If you’ve given up wading through the impenetrable English of the report, I’ll summarise the paper for you. There are areas of intellectual endeavour we can confidently label as “science”. All the rest, normal people call “art”.

    Pointman

  31. Good to see Hayek referenced at all. See also Wisdom of Crowds and Army of Davids.

    Judy and Muller have both mentioned that they just trusted what they were told by other scientists until it became obvious that what they had been told was not true. The so-called “consensus” couldn’t exist without this trust. And when the details of so much of the ‘science’ is finally audited and replicated, when the data is made available, the FOIA requests complied with, the lack of quality control exposed, when the stats are corrected and the software brought up to quality standards, we will finally be able to see what is really known and how much was merely hope, trust and confirmation bias.

    Glad to see that everyone is beginning to understand why trust has no place in science. It’s all about the verify. Audit, replicate, check the stats, fix the software. Models aren’t worth anything until they’ve been verified and validated.

  32. Knowledge is hidden? They’ll not be able to tease if from my cold dead synapses so this is the best I can do: We are cooling, folks; for how long even kim doesn’t know.
    ==============

    • Hmmm,
      Electric shock treatments before the synapses go haywire?
      Na, that would just produce pain…. :-)

    • Knowledge is hidden in physical science because it is a process of discovery of facts. This is not true for economic science, which is a process of invention of processes.

      • Do not forget about the hidden or concentrated knowledge that Monopolies or more prevalent Oligopolies rely on to coerce markets. Derivatives instantly come to mind. The Housing bust was largely a part of Derivative Fund managers having special or hidden knowledge that a fund they insured elsewhere was going to fail. There is a big difference in distributive and ‘hidden’ knowledge. Hidden means available to some and not to most.

      • Derivatives did not cause “the housing bust.” They merely spread the damage from the bursting of the bubble.

        Goldman Sachs is the best example of hidden knowledge in this sense, since they were shorting the very mortgage derivatives for which they were creating a market. But as to the point of this thread, it was not that knowledge that saved them, it was the central planners of the Obama administration. If they had not intervened by funneling billions of TARP funds to Goldman, through AIG, Goldman would have suffered the consequences of their decisions, just as Lehman Brothers did.

        The survival of Goldman Sachs was not the free market or hidden information in action. It was rent seeking “crony capitalism” at its worst.

  33. Prices provide information to those working in, creating and managing the systems of production. There is no equivalent in the system that produces climate forecasts. An inaccurate forecast is indistinguishable from an accurate one.

    • Prices are the end result of the free market system’s means of processing information. They don’t necessarily provide individual participants the detailed information that results in a given price (as Hayek made clear in the article), but represent an accumulation of the knowledge of the market as a whole. Climate forecasts are by no means excluded from the principles Hayek discussed.

      Climate forecasts are simply additional information that is available to the participants in the market. They are in their function not different from economic forecasts, commodity forecasts, debt reliability forecasts, etc. If climate forecasts were simply an academic exercise, there would be far fewer followers of this and other climate blogs. It is their economic and political impact that makes them so important. The “equivalent system” for those forecasts then is (or at least should be) that same free market.

      The problems arise when progressives control the funding (and thus limit the direction) of research, and when they use the resulting climate forecasts to justify central economic planning. The free market has already produced forecasts and other knowledge that essentially compete with (by contradicting) the government subsidized consensus. That is what the fuss is all about.

      Put another way, a free market will not make individual climate forecasts more or less accurate. But it will make it more likely that all forecasts will be subject to competition, increasing the total knowledge of the market as a whole. A free market will also benefit from the total of the knowledge of all of its participants, which is therefore far more likely to ultimately guide the market to appropriate responses to those forecasts.

      • What a market will do is prevent a particular socially favored forecast from becoming dominant. It won’t pick the “right” one out of the noise any better than it will pick better performing mutual funds. Remember, index funds usually outperform managed funds.

        What it will do is minimize the chances of the scientific equivalent of a Madoff fund from sucking too many people in.

      • I would put it a little differently. A free market would not necessarily minimize the number of people sucked into a “Madoff fund,” it would simply ensure that there were other competing funds so that when the Madoff scheme collapses, it does not take the entire economy with it.

        This is one of the biggest problems with centrally planned economies. When the government controls an industry, like the East Germans controlled their auto industry, all you get are Trabants, which Time Magazine dubbed the worst car of all time. The problem is not faulty climate forecasts, it is the risk that central planners will be able to use them to force us all into one grand CAGW/Madoff Climate Fund.

      • “Minimize” wasn’t the right word. We’re in full agreement. The key point being that competing pursuits of the truth won’t necessarily result in the truth, which may be unknowable. What they will do is limit the number of people who get sucked into the whirlpool of epic wrongness. Avoiding that whirlpool may be the best we can hope for.

        Hidden knowledge may be getting more credit than it deserves; nothing guarantees that what these distributed systems bubble up is anywhere close to correct or optimal. Survival of the fittest has more to do with survival than fitness.

      • Gary,

        I think I agree with your assessment on the function of climate models with respect to informing decision makers. They provide information that could be useful for understanding which decisions are good or bad to make about the future. In that sense, they share something with economic forecasts, although the underlying principles of a climate model are much better understood.

        I’m having a hard time understanding some of the distinctions implicit in this description.

        ‘The problems arise when progressives control the funding (and thus limit the direction) of research, and when they use the resulting climate forecasts to justify central economic planning.’

        First, there is no evidence to support this claim. The last two Democratic presidents have been just as happy to hand over the reigns of economics planning to private sector as any Republican president in the past 30 years. So I challenge the factual accuracy of such a statement.

        But even if we assume such a premise is factually correct, I still don’t understand the distinction in practical terms. The practical alternative according to the neo-conservative ideology is that the ‘free market’ decides economic planning, as you have espoused above. In the past 30 years, however, the ‘market’ has centralized itself to a handful of very large, very powerful firms who claim ultimate understanding of the economy, but cannot prove such an understanding with any reproducibility. Even in the face of economic disaster which they had told the public would not occur. Such claims also fail the tests above for many of the same reasons understanding of the climate has evaded us.

        So while you, and possibly many others here believe, that the centralizing decision making is bad, especially concerning economic decision making, the ‘free market’ has done just that.

        So why is it more practical to have private centralized economic decision making driven by the ‘free market’ versus public centralized decision making? If centralized decision making is bad, why are we promoting an ideology that has lead directly to centralized decision making? Centralized decision making that has cost the world some 10’s of trillions of dollars of wealth in the last 5 years.

        I don’t understand that point.

      • maxwell:

        “The practical alternative according to the neo-conservative ideology is that the ‘free market’ decides economic planning, as you have espoused above.” I think that this sentence shows where you do not understand what conservatives mean by “the free market.”

        “[T]he free market decides economic planning” is an incorrect way of putting it. The choice is not between an economy planned by the government, and an economy planned by the market. A free market economy is not planned. It is a system that allows all individuals to make their own economic decisions without being constricted by any central plan.

        The issue of large corporations being the equivalent of governmental central planning has some merit, but it is a problem the market deals with regularly. IBM, General Motors, Chrysler, AIG, Lehman Brothers, all were huge corporations that were severely disciplined by the market for their inability to adjust to economic realities. Some learned to adapt, others failed and entered bankruptcy. And in the worst case, the central planners took over and forced the manufacture of a car no one wants, at a price no one will pay.

        As to your comment that corporate “centralized decision making … has cost the world some 10′s of trillions of dollars of wealth in the last 5 years…,” those bad corporate decisions were a direct response to manipulations of the market by progressive central planners. It was the distortion of the free market by the government that caused the massive economic losses of the last several years.

        The entire sub-prime home mortgage market was a creation of the U.S. Congress, aided and abetted by the quasi-governmental Fannie Mae and Freddie Mac. Banks refused all the progressives’ attempts to force them to make bad loans until the government began to require such loans, and provided a government guaranteed mechanism for buying the poor quality loans once they were generated.

        By the way, there is nothing “neo-conservative” about free market theory, it is just conservative. Hayek was not prescribing a plan for future economists to follow based on his own insights. He was describing the way out current system evolved, and counseling against the New Deal’s attempts to dismantle it. That is the core of economic conservatism, to preserve the accumulated knowledge and traditions that have created the strongest economy the world has ever seen.

      • Gary,

        ‘The choice is not between an economy planned by the government, and an economy planned by the market.

        Here we are. After 50 years of hearing about the benefits of the ‘free market’. After 30 years of trying the ‘free market’. And that is the practical choice we have. The ‘free market’ ideology might say something different, I agree, but we find ourselves with a completely different situation in practice.

        And that is the choice. Public central planning versus private central planning. There is no arguing with the real world.

        ‘…those bad corporate decisions were a direct response to manipulations of the market by progressive central planners.’

        You would characterize Reagan as a ‘progressive’? More than that, market control via regulation had worked to stem off large crashes like this latest one for the 50 years between 1929 and 1979. Regulation that put into place by ‘progressives’. So such an assertion flat out wrong.

        It was the lifting of regulations to allow the ‘free market’ to flourish that created the large risks leveraged against debt to be mounted. As well as the continuous fight the ‘free market’ adherents against further regulation of market ‘innovations’ produced over the last 20 years.

        Also,

        ‘…And in the worst case, the central planners took over…

        Yes, conservative central planners who had recently been running the very companies that fought regulation and championed ‘free market’ ideas. The original TARP legislation was the brain-child of Bush and Paulson. Not ‘progressives’.

        …until the government began to require such loans, and provided a government guaranteed mechanism for buying the poor quality loans once they were generated.

        Yup, under the auspices of the ‘free market’ mantra in which regulation of the roles of financial institutions were lifted so that those poor loans became not only a commodity that could traded, but whose default could be bet against. I have yet to see some legislative regulation that designed sub-prime loans so that they could bought and traded as a commodity. That was a ‘free market innovation’ as far as I understood.

        I am willing to be proven wrong with some documentation, however.

        Finally, this is best one,

        …to preserve the accumulated knowledge and traditions that have created the strongest economy the world has ever seen.

        Traditions that include social security, Medicare, welfare assistance, a progressive tax and regulation that protects the entire economy from risks that benefit few and hurt the majority? Because at our peak, the US was a highly regulated economy that used financial assistance from the government to spur some of the greatest technological advancements in human civilization.

        Here’s the rub for me on conservatism. I think that at its core, an ideology in which each person acts in his/her own interests, thus creating a good world for everyone is great. We then individually identify the best producers and products, thus stabilizing a system in which good work is rewarded and multiplies.

        Unfortunately, some groups of people can very easily manipulate such a system for their personal gain, as we’ve seen again and again and again in the past 30 years.

        So while I am drawn to a philosophy that looks great on paper, the actual data that has been coming in since ‘free market’ principles have been implemented in the real world does not look good. The income gap is widening. The median income is down. Home ownership is down. Foreclosures are up. And we’re getting our collective butts kicked in innovation.

        So again, I don’t understand all the hype. The data seems to tell a different story.

      • maxwell,

        What precisely are the regulations that you think were repealed to allow banks to make terrible high risk loans? I know “deregulation” is the rallying cry of Barney Frank, Chris Dodd and the other progressives who combined to force banks to make these bad loans. But other than repeating their mantra, do you have any “data [that] seems to tell a different story?”

        I don’t have the time to explain the entire history of the development of capitalism to you. Nor is this blog the place to debate the entire progressive/conservative economic debate. My comments have been about the utility of the free market in the context of this thread regarding climate research and policy.

        But as an illustrative point, if you can actually come up with a regulation that you believe prevented the making of high risk loans that was later removed by the evil deregulators, I would be glad to respond.

        I have previously listed on a different thread the long list of regulatory and statutory changes that created the high risk home loan market. They began with the Community Reinvestment Act, and the process concluded with the Congress requiring Fannie and Freddie to purchase high risk loans from the originating lenders. By effectively mandating such loans, and then severing the underwriting of the loans (ie. the decision to make them) from the risk of loss, the government virtually guaranteed a crash.

      • Gary – GogglenThe Commodity Futures Modernization Act (CFMA),– which ws the “United States federal legislation that officially ensured the deregulation of financial products known as over-the-counter derivatives.”

        Here’s a nice snippet from an overview:

        http://www.scribd.com/doc/34013485/The-Role-of-Derivatives-in-the-Financial-Crisis

        By removing the multi-trillion dollar swaps market from the traditional norms of market regulation, a highly speculative derivative bubble was created that was opaque to federal regulators and market observers alike.By removing all forms of ensuring the normal capital adequacy protections of market regulation, the swaps market permitted trillions of dollars of financial commitments to be made with no assurance that those commitments could be fulfilled beyond the highly illusory AAA ratings of the counterparties in question. Had the norms of market regulation been applicable, these swaps transactions would have been adequately capitalized by traditional clearing norms; and the dangers building up in these markets would otherwise have been observable by the transparency and price discipline that accompanies exchange trading.

        By virtue of no capital adequacy requirements, we got leveraging of 40 – 1 by speculators, thereby multiplying the impact, many times over, of their systematic underestimation of the risk involved with their massive investments.

      • I should have added: “…..their massive investments in bad mortgages.”

      • Joshua,

        The CFMA is not an example of deregulation. New financial derivatives at the time were not subjected to the regulations of other financial instruments. Before the CFMA, many derivative trades were being conducted over the counter, not through the exchanges, and were among sophisticated investors, who could (and many did) conduct the transactions overseas. Since the trades were not being conducted on the U.S. exchanges, they were not regulated at all.

        The CFMA was an attempt to bring those derivatives under some manner of regulation. In short, the Act subjected to regulation transactions that had until then been made outside the existing regulatory regimes.

        Moreover, the article you cited is not a serious analysis of the housing meltdown, it is an attempt to deflect responsibility from the progressives who caused it. The author, Michael Greeberger, a one time member of the Clinton Justice Department, has an interesting web page.
        http://www.michaelgreenberger.com/
        What I find most interesting is his choice of a photograph of George Soros and himself testifying before Congress for that home page.

        In his article, Grenberger made the typical “everybody knows” claim, so popular among progressives to support their positions. But the economists he cites for this “consensus” are Paul Krugman (resident economics demagogue of the N.Y. Times), Joseph Stiglitz (a former Clinton adviser), Alan S. Blinder (another Clinton adviser), and James K. Galbraith (the intellectually faithful son of the father of modern central planning, John K. Galbraith), progressives all.

        Not a single free market supporter among them.

        Derivatives did not cause the housing crash. Essentially, many of the complex mortgage backed securities involved in the crash were pieces of bundles of poorly underwritten mortgages. When the mortgages began to default, and the market responded by lowering the price of those securities, whoever was stuck holding them got burned. But the mortgage losses would have been there, with or without the derivatives.

        Complex derivatives did create other risks, and losses, for those who invested in them, but they were not the cause of the housing meltdown. Greeneberger notes toward the end of his article that similar securities, backed by sounder goods, are still on the market: “The securitization structure (including CDS) is present not only in the subprime mortgage market, but in the prime mortgage market, as well as in commercial real estate, credit card debt, and auto and student loans.”

        All of the complaints Greenberger makes about regulation and capitalization of sub-prime derivatives can also be made about these other securities. Yet none of those derivatives were part of the meltdown. Again, it was not the nature of the derivatives, but of the underlying good, that caused the crash.

      • The CFMA is not an example of deregulation

        Gary – you can’t be serious.

        In 2004, the Securities and Exchange Commission relaxed the net capital rule, which enabled investment banks to substantially increase the level of debt they were taking on, fueling the growth in mortgage-backed securities supporting subprime mortgages. The SEC has conceded that self-regulation of investment banks contributed to the crisis.[133][134]

        Financial institutions in the shadow banking system are not subject to the same regulation as depository banks, allowing them to assume additional debt obligations relative to their financial cushion or capital base.[118] This was the case despite the Long-Term Capital Management debacle in 1998, where a highly-leveraged shadow institution failed with systemic implications.

        […]

        The U.S. Congress allowed the self-regulation of the derivatives market when it passed the Commodity Futures Modernization Act of 2000. Derivatives such as credit default swaps (CDS) can be used to hedge or speculate against particular credit risks. The volume of CDS outstanding increased 100-fold from 1998 to 2008, with estimates of the debt covered by CDS contracts, as of November 2008, ranging from US$33 to $47 trillion. Total over-the-counter (OTC) derivative notional value rose to $683 trillion by June 2008.[138] Warren Buffett famously referred to derivatives as “financial weapons of mass destruction” in early 2003.[139][140]

        Author Roger Lowenstein summarized some of the regulatory problems that caused the crisis in November 2009: “1) Mortgage regulation was too lax and in some cases nonexistent; 2) Capital requirements for banks were too low; 3) Trading in derivatives such as credit default swaps posed giant, unseen risks; 4) Credit ratings on structured securities such as collateralized-debt obligations were deeply flawed; 5) Bankers were moved to take on risk by excessive pay packages; 6) The government’s response to the crash also created, or exacerbated, moral hazard. Markets now expect that big banks won’t be allowed to fail, weakening the incentives of investors to discipline big banks and keep them from piling up too many risky assets again.

      • . But the mortgage losses would have been there, with or without the derivatives.

        Resulting from various forms of deregulation, banks leveraged 40 – 1 to invest in CDOs. That is what created a global financial crisis when the housing bubble burst. Banks and hedge funds (with a few notable exceptions that did not suffer nearly as much when the crisis occurred) systematically underevaluated the risk of hedging the stock market through derivatives. It is well documented that they relied on econometric formulas that improperly calculated the risks. The financial crisis resulted from failed risk management on a massive scale by massive financial institutions.

      • And btw – there is some very interesting implications of the failure of econometric computer modeling to climate science.

        Recipe for Disaster: The Formula That Killed Wall Street
        http://www.wired.com/print/techbiz/it/magazine/17-03/wp_quant

        How computer modeling worsened the financial crisis and what we ought to do about it
        http://www.theamericanscholar.org/the-terminator-comes-to-wall-street/

        How Wall Street Quants Lied to Their Computers:
        http://bits.blogs.nytimes.com/2
        008/09/18/how-wall-streets-quants-lied-to-their-computers/

        Gaussian Copula Function
        http://superfantastik.org/minerva/?p=385

      • This is good, also:

        Since the onset of the economic crisis economists have been criticised, with rare exceptions, for having failed to predict this turn of events. The purpose of this talk is to argue that not having forecast the onset is excusable but having systematically built macroeconomic models in which such an event could not happen is not excusable. The path that we followed to this point, from Walras to the present has been one which was based on an inappropriate notion of equilibrium and a narrow focus on “efficiency”. Furthermore, the theoretical role attributed to markets does not correspond to how they function in reality. Poincare already pointed this out at the time of Bachelier’s thesis, but his message went unheard. If we are to be able to understand the workings of the economy and to be able to give useful advice, we have to look at it as an interactive system, to analyse the network structure of the interaction between individuals and institutions and to focus on how economy is coordinated. This does not imply abandoning analysis but does mean creating models with different mathematical structures than those to which we are accustomed. We cannot be pardoned for vaunting the merits of models to policymakers when those models are so obviously deficient.

        […]

        Economists not only failed to anticipate the financial crisis; they may have contributed to it – with risk and derivative models that, through spurious precision and untested theoretical assumptions, encouraged policy makers and market participants to see more stability and risk sharing than was actually present. Moreover, once the crisis occurred, it was met with incomprehension by most economists because of models that, on the one hand, downplay the possibility that economic actors may exhibit highly interactive behavior; and, on the other hand, assume that any homogeneity will involve economic actors sharing the economist’s own putatively correct model of the economy, so that error can stem only from an exogenous shock. The financial crisis presents both an ethical and an intellectual challenge to economics, and an opportunity to reform its study by grounding it more solidly in reality.

        http://www.econometica.it/events/2009/30-11/prof_Kirman.pdf

      • OK, one more that I just found: This is explained in such a thorough and systematic manner, even I could understand:

        Are Mathematical Models the Cause for Financial Crisis in the Global Economy?
        Andrew W. Lo, director of the Massachusetts Institute of Technologys Laboratory for Financial Engineering

      • Joshua,

        Please don’t cite long passages from Wikipedia as “evidence.” It’s a waste of time. Wikipedia is no more reliable on economic issues than it is on climate. Here is a quote from the Wiki/progressive article you find so illuminating: “Another analysis, different from the mainstream explanation, is that the financial crisis is merely a symptom of another, deeper crisis, which is a systemic crisis of capitalism itself.”

        Wow, progressives blame their destruction of the housing market on capitalism. Who woulda thunk it?

        As for the CFMA, nothing you quoted disputes what I actually wrote. I’ll type this slowly for you so you can follow:

        The derivatives you are so certain were the cause of the financial crisis were not DEregulated. They were for the most part never regulated at all from their creation. They started out “self-regulated,” which I explained above.

        I do find your cutting and pasting of criticism of economic models amusing. Economists (and their models) are no better at predicting the future than climate scientists (and their models). This is no news to conservatives. But coming from such a fan of central planning as yourself, it is simply too precious. I anxiously await your next condemnation of attempts to decide public policy based on climate models’ predictions of CAGW.

      • Gary. Please re-read what I have and have not said:

        The derivatives you are so certain were the cause of the financial crisis were not DEregulated

        Or perhaps I should be more clear. I never said that deregulation of securities “caused” the financial crisis. To the extent that I suggested a single cause, I pointed to the failure of financial institutions to manage their risk as they leveraged themselves up to 40 – 1 to buy bundled packages of bad debt (and to “insure” that debt with institutions that lacked the sufficient collateral). But there were many causal factors that contributed to the financial crisis, and a lack of regulation of securities, which enabled the explosion in unregulated investments in bad debt, was one of the causes.

        As for your point about derivatives always being deregulated. The point is that the CFMA prevented regulation of the types of CDOs which contributed, massively, to the financial crisis:

        –snip–

        In 1998 the Commodity Futures Trading Commission called for regulation of the futures market, which included credit default swaps, however, Chairman Greenspan, along with the Secretary of the Treasury Rubin and the Economic Advisor to the president, Summers were all against such actions, and thus it remained unregulated

        –snip–

        I quote Phil Gramm:

        “The work of this Congress will be seen as a watershed where we turned away from an outmoded Depression-era approach to financial regulation and adopted a framework that will position our financial services industry to be world leaders into the new century,”

        A pretty interesting quote given that Gramm later said that deregulation had nothing to do with the financial crisis.

      • Joshua,
        I amlargely in agreement with you on the derivative debacle.
        I traded CMO’s for a few years and had a lot of fun and made money with them.
        There were amazing yields and gains to be had from them, if you bought carefully and excercised prudence and patience.
        I left about the time Orange county blew itself up by trading derivatives on margin- I could not believe anyone in their right mind would buy them on margin.
        Since those mid 1990’s years, the derivative markets got crazier, and the leverage was not mere margin of 2:1, but, as you point out, ridiculous levels of 40:1 in some cases.
        My main regret is that I did not merely step out of the market in 2007 when Bear sterns imploded, but did not massively short the entire market.

      • hunter –

        Once we get past the partisan bickering about whether regulation or the lack thereof was causal, we are left looking at the more basic mechanics of what caused the crisis. Certainly, the ubiquitous sub-prime loans were one of those mechanics (and again, we could bicker about who is responsible for that, the CRA – even though there was significantly less sub-prime lending associated with the CRA – the home-owners themselves, the mortgage brokers, etc). But the irresponsible discounting of risk by financial institutions, and the irresponsible leveraging they used to buy and sell bad debt (which multiplied the impact of mortgage defaults exponentially, and concentrated them on a few entities which controlled some 40%? of our GDP) were obviously key factors. Anyone who tries to ignore that reality is kidding themselves.

      • “Once we get past the partisan bickering about whether regulation or the lack thereof was causal, we are left looking at the more basic mechanics of what caused the crisis.”

        A classic Joshua post. Joshua inserts himself into a discussion (which is fine by the way, this is a blog after all) between maxwell and myself in which I asked the following question:

        ‘What precisely are the regulations that you think were repealed to allow banks to make terrible high risk loans?”

        Joshua entered the conversation with the following comment:

        “Gary – GogglenThe Commodity Futures Modernization Act (CFMA),– which ws the ‘United States federal legislation that officially ensured the deregulation of financial products known as over-the-counter derivatives.'”

        Followed later by:

        ”The CFMA is not an example of deregulation

        Gary – you can’t be serious.”

        So after bickering ad nauseum, with extensive quotes pasted from Wikipedia and other progressive opinion pieces, Joshua now counsels against “partisan bickering.”

        I won’t bother addressing what others have to say on the financial crisis, I have read numerous progressive rationalizations which ignore the impact of their own legislation, , regulation, “community organizing” acts of black mailing banks into making loans, and forcing Fannie and Freddie to purchase sub-prime loans while providing implicit government guarantees.

        I will end by simply repeating:

        The CFMA was not an example of deregulation, which was the question Joshua responded to.

        de·reg·u·la·tion noun \(ˌ)dē-ˌre-gyə-ˈlā-shən\ the act or process of removing restrictions and regulations.

        Derivatives were and are a risky investment that should only have been entered into by the most sophisticated investors. The huge losses on derivatives that resulted from the hubris of the self-appointed financial elite are not in question. It is a common problem among elites to overestimate their own intellectual prowess. But those derivatives did not cause the sub-prime crisis, nor did their supposed deregulation.

        Progressive’s reflexive need to defend all their prior bad decisions and actions no matter how obvious the errors gets tiresome. The sub-prime market was created, intentionally, by congressional progressives and their partners in the regulatory and “community activist” communities. No amount of revisionism can change that.

      • Pricing forecasts.
        Weather forecast. A series of forecasts can be recorded and compared to actual results over the forecast periods – a number of days.

        Sales/market forecast. A series of forecasts can be recorded and compared to actual results over the forecast periods – a number of months or years.

        Climate forecast. A climate forecast can be recorded and compared to actual results over decades or centuries.

        People price weather forecasts based on historical accuracy. People price sales and market forecasts based on historical accuracy. People will price climate forecasts at some point in the distant future when they are able to evaluate their historical accuracy. Currently, climate forecasts are rent seeking academic exercises for which no real market exists.

      • I guess all those people who have voted for cap and trade systems, and created markets to engage in carbon trading, and invested in “green technologies,” and spent huge sums on “green advertising,” didn’t get the memo that they were supposed to wait.

        It would be correct to say that people SHOULD “price climate forecasts as some point in the future.” But it is a mistake to think that they are actually waiting to do so. (Not to mention, climate scientists have been making such forecasts at least since the 70s, which I think qualifies as “decades.”)

      • GaryM
        The Waxman-Markey bill (Cap and Trade) failed to pass the US Congress. To the best of my knowledge there has been no nationwide US referendum on Cap and Trade.

        Investments in “Green Technologies” are largely rent seekers requiring tax and regulatory support – other peoples’ money. In any case, neither Cap and Trade nor investments in Green Technologies represent cash payments to providers of climate forecasts. No cash, no price.

      • “People price weather forecasts based on historical accuracy. ”

        I’m not sure you get the point of my comment, or the above article. The issue is not “pricing climate forecasts.” Neither I nor anyone else as far as I have read has made any such suggestion.

        My comment was: “Climate forecasts are simply additional information that is available to the participants in the market. They are in their function not different from economic forecasts, commodity forecasts, debt reliability forecasts, etc.”

        “Pricing” in this context does not refer to the price of any particular good or service, but the mechanism by which a market makes use of what is being referred to as “hidden knowledge” (a term I am not terribly fond of, as “hidden” suggests some sort of intent). The information being provided to date by climate forecasts does not have much current value (though you are wrong in thinking no one is paying for those services), but the issue of my comment was what type of system is most likely to increase the accuracy of such forecasts, best utilize whatever information they do provide, and correct for the inevitable errors.

        You seem to think I am somehow defending the current value of climate forecasts. You are mistaken.

  34. Judith,

    You keep wanting to distinguish what is certain and what is uncertain.
    Do we know for certain how a planet is created?
    Do we know how much water has been lost since the formation of this planet?

    In order for a planet to form, it has to stop the loss of mass from the forces of centrifugal force. To do this, a shell has to be created to contain this mass.

    There has been a great deal in science that has made me scratch my head at “how did they come to this conclusion?”
    I have done a great deal of research into Ice Ages and what was before that. So far ALL the research is pointing that this planet had at least 10 times more water volume of oceans.
    Why was there no plant and animal life before 500 million years ago out of 4.5 billion? Why does carbon dating give us different time lines in rocks being formed? You cannot carbon date objects that were not being exposed to air. Our ocean core sample dating is through the accumulation of H2 18 O in the shells of animals that have died.
    The conclusion of Ice Ages shaping our landscape and making rock is very faulty as the pound per square inch of ice is too light to that of the same volume of water. Why is that? Water collapses on itself to a point in a cone shape making pressure. Ice is too porous and objects melted around it will NOT feel any pressure.

    Take sand. Abundant all around this planet on the surface and in the oceans but how was sand made? Theory is through water hammering rocks over the billions or rainwater. I have yet to see rain create a grain of sand unless it is on a rock already compressed from sand.
    But, volcanic activity creates porous rocks under immense pressure these can implode due to pressure and trapped gases. Some very old volcanic rock is highly compressed and not porous like today’s volcanoes, why?

  35. enjoyed the post

  36. The price analogy helps explain the boom-and-bust history of scientific ideas. Despite scientific method being self correcting, science is a highly distributed social system. It is just as prone to fads as any other. Moreover, even good ideas run their course, to be replaced by better ideas. Simple AGW boomed, let’s hope it is now crashing due to a more complex view of climate.

    • And sometimes old discredited ideas continue to be useful models, too. We still talk about the sun coming up in the morning, for example. We don’t take QM and relativity into account when designing airplanes. They still fly.

      The trick is knowing what matters. That’s what everybody’s tripping over.

  37. The idea of “science beyond individual understanding” is an invitation to surrender your mind and soul to “Big Brother.”

    Eisenhower warned of the potential danger to our free society from government-directed mob mentality in science in this portion of his farewell address to the nation on 17 Jan 1961.

  38. Could someone help me with a simple question? One of the fingerprints of AGW frequently mentioned is the fact that there has been a greater increase in the nighttime minimum temperature than the daytime maximum temperature. What is the physics behind this reasoning? The earth’s surface specific heat doesn’t change, and the returning radiation from the greenhouse gasses would be expected to be less at night. I downloaded the data from the USHCN and the average of all of the stations and an area-weighted average by state both show virtual no increase in the maximum temperature and a .6-.7 degree per century increase in the minimum temperature.

    • B,

      when the sun is down, there is no incident energy on the earth’s surface. That allows the objects on the surface as well as the lower levels of the atmosphere to cool substantially, especially on clear nights. This cooling happens via emission of long-wave IR light, some of which is in the region of absorption by GHG molecules. As more and more GHG’s are added to the atmosphere, less and less of this upwelling long-wave IR light can escape directly to outer space. This insulation keeps the nighttime warmer than without more GHG molecules.

      During the day, the surface of the earth and lower levels of the atmosphere are still emitting long-wave IR light, but the total flux of energy on the earth is conflated by the sun. So an effect to daytime temps from increased GHG’s could easily get lost in the noise, especially if we don’t have a good multi-decadal model for the sun, which I think is true.

      • I was looking for a distinction on the physics that is happening. “the total flux of energy on the earth is conflated by the sun”?? “get lost in the noise”?? Radiation is radiation. Specific heat is specific heat. There is also a conservation of energy rule. What changes at night?

      • B,

        the physics is exactly the same during the daytime and nighttime. The earth is constantly emitting radiation at a peak energy and rate proportional to its temperature. The difference between night and day is the sun. Because there is no sun light coming in during the night, the effect of extra GHG molecules in the atmosphere is easier to pick out from the noise in the actual data.

        But there is no ‘distinction on the physics that is happening’. It’s a distinction is our ability to measure an effect given a specific background. That is, sunlight vs. no sunlight.

      • I’m sorry, I don’t buy the “noise” explantion. In engineering, we call that “hand-waving”. The nighttime warming is presented as a basic arguement for AGW. I am looking for a sounder explanation.

      • B,

        As far as ‘noise’ is concerned, look at the data and calculate the standard error in the daytime temps and the nighttime temps. If the nighttime standard error isn’t larger than the daytime standard error, then you can come back and prove me wrong. Until then, you’re the only one ‘hand-waving’.

        More importantly,

        ‘I am looking for a sounder explanation.’

        What exactly isn’t ‘sound’ about my explanation? That you don’t like it?

        There are several threads on this blog that go into gorey detail on all the physics behind the greenhouse effect and how it is affected by the presence of the sun. I suggest you inform yourself a bit more before you decide which explanation is ‘sound’ or not.

        I think you’re conflating your understanding of this problem with my explanation. I have explained all the basic physics involved. If you don’t understand how it works, then you need to think about it a bit harder about it.

      • The nighttime standard error is 1.53 degrees, the daytime standard error is 1.65 degrees. Any other thoughts?

      • There is a significant additional factor: convection.

        When sun is heating the ground, convection is increased and the warming extends to a significant height in the troposphere. Basically we are close to the situation, where the warming maintains the adiabatic lapse rate. During a calm clear night the radiative balance leads to a relatively rapid cooling of the surface and the very lowest troposphere. Convection stops or is at least reduced and radiative heat transfer is the only important effect. Under these conditions the surface temperature is more sensitive to the GHG concentration than during the day, when convection is important.

      • B,

        that’s great. So you confirmed the hypothesis. The ‘noise’ in the nighttime signal is smaller.

        You’re welcome.

      • M: You’re saying that a n 8% change in standard error confirms your hypothesis? Interesting.
        P.P. Good explanation, but I would think that a noticeable warming would still be measureable during the day. Is there data available of measurements comparing daytime(excluding solar) and nighttime IR?

      • Pekka, if that were true, the weather would be calmer at night than during the day. All weather is really natural convection, and all convection is really atmospheric disturbance. Is there any evidence of this?

      • ChE,
        I didn’t write “During a clam clear night ..” by accident. Not every night and not everywhere is like that, but they are common enough to have a significant effect on the averages.

        Is there evidence that the temperature variations are larger during calm clear nights? There certainly is at it’s very clear at high latitudes (like here in Finland), at high altitudes and also in deserts everywhere. It’s quite common that a temperature inversion develops during the night. A rapid 10 degree temperature variation builds up easily during such conditions.

        Are such variations important indicators of the global warming, or are they misleading numbers influencing the artificial concept of global average surface temperature more than the more relevant global heat content, is another issue.

      • Thanks all of you for your interesting and informative replies to my question. But as my jr. high school science teacher used to say, “The more you know, the more you don’t know.” Does the mixing during the day mean that there is more heating than is measured, or is the temperature of the the well-mixed air a better indicator of the actual heating? Conversely, does the measurement of the stratified air during still evenings distort the GHG impact? Under these conditions, wouldn’t the impact of ground use changes and the dreaded UHI effect be more apparent? Thankfully, nobody suggested changes to blackbody radiation at night when everything looks black. I will continue to pursue these questions on other avenues.

      • PP;
        So, “a clam, clear night was deliberate? What’s a clam night like?

        Please reply in a clam, clue, and corrected manner.

      • Given Ryan Maue’s chart of low recent Accumulated Cyclone Energy I’ve asked Chris Mooney when he’s going to write ‘Clam World’. Some of the effluent at his site made him bilious though, I believe, and he hasn’t oystered up any pearls.
        ======================

    • With the mass of concrete and asphalt and other heat retaining materials humans continually add to the environment, it isn’t particularly surprising to me the nighttime temperatures might show a modest increase over time. Those materials aren’t able to cool as effectively at night as the natural environment.

      I suspect the poor siting of thermometers near structures doesn’t allow readings to fall as low as might be measured further from structures and asphalt.

      It nighttime increase is likely anthropogenic, but not CO2 related.

      That’s my two cents. But I don’t claim to be a scientist.

  39. I am a little of shocked that both the blog entry and the comments that follow it demonstrate so little understanding of ‘the knowledge problem’, and how important it is to physical science.

    Karl Popper (who wrote the Logic of Scientific Discovery) and Thomas Kuhn (who wrote the structure of scientific revolutions) approached the problem of knowledge in the domain of DISCOVERY, which we call the physical sciences. At the same time, the CLOSE FRIEND of Karl Popper, Friedrich Hayek, worked on the problem of hubris in the social sciences. Hayek ended up combating Keynes over the frailty of models and reason. Keynes wrote ‘A Treatise On Probability” and then the “General Theory” which led to the governmental use of economic calculation that all of us live under, by trying to solve for unemployment – something Hayek (correctly) stated was not possible in the long run and would lead (as it has) to our bankruptcy. Hayek stated that traditional knowledge that was handed down, and perpetuated by trial and error, was ‘true’ even if we did not understand it rationally as yet. And that our record of rational judgemet was exceedingly frail, and that history was filled with examples (past and present) of ridiculous scientific error. Keynes won in the short term however, because his theory solved the problem of socialism by replacing a false pretense of REASON (a managed economy), with the false pretense of PROBABILITY ( a monetarist economy) in the field of national economics. In other words, Keynes gave politicians power over our economies. The power that has led us to our financial crisis.

    BOTH Popper and Hayek were countering two problems. A) the use of the fairly new field of statistical analysis and it’s limitations at prediction due to what Nassim Taleb calls the LUDIC FALLACY. And B) the rise of Socialism, and the socialist hubris of central planning . These men, plus Hayek’s mentor Ludwig von Mises, effectively undermined and predicted the impossibility of a socialist economy. Both men stated that human minds are frail and capable of very limited reason and prediction. More importantly, that the Anglo RECORD OF WHAT WORKS, or EMPIRICISM, is a superior form of KNOWLEDGE to the French (and then Marxian) fantasy of RATIONALISM. The fact that the average american does not understand this by doctrinare education difference is probably equal to the use of today’s mysticism in Islam or medieval christianity. It is a means of keeping people ignorant. We have attempted to replace mysticism with science without also teaching history (mythology) which teaches us the error of hubris. We do not teach history because in a pluralistic society, history includes value judgments and value judgements are class, race, and culture judgements. However, aside from class, race and culture, we are taught only the error of the churches, without the errors of silly scientists who were little better than shaman. We did not teach our children HUBRIS. Greek mythology teaches one lesson above all others: Hubris.

    To the physical sciences, which is the process of DISCOVERY of what EXISTS already, is the objective of study. The holy grail is to discover the first-causal properties of the physical universe. To economists, the problem is one of INVENTION. This is called Hume’s problem, or the problem of induction. That is, what can humans INVENT given their current state of knowledge. THe problem of economic science, which is the ONLY social science we yet possess, is similar to climate science in complexity, yet additionally more complicated because there is no process of EQUILIBRATION in the intellectual world. (there is no human equivalent to the law of thermodynamics – there is in fact, energy added to the system. We call it ‘increases in production’.) Nor is time constant. In fact, that’s what productivity does: it creates more ‘time’ by using less of it to produce more calories. Instead, of an equilibrium as in nature, the mind of man invents new ideas all the time from permutations of existing patterns and disrupts all attempts at equilibria.

    Both forms of our theories, whether physical science or economic science, can only be tested by FALSIFICATION. Unless you can stipulate ACTIONS by which we can prove climate hypothesis false, they are not in fact, scientific. For example, Einstein said that the absence of red shift would falsify one of his theories.

    The first principle of Greek Rationalism is SKEPTICISM as a warning against HUBRIS. Hubris is a danger because of the cognitive biases humans of necessity possess because we attribute higher value to that which we study most.

    TRUTH IN SCIENCE IS PREDICTION AND FALSIFICATION. Models are not truth. They are tools for rationalizing data. The carpenter may not understand the metallurgy of his chisel. He may not understand the distribution channel for his wood. The scientist is often using chisels and wood that he or she does not understand. If he or she understood, then he or she would understand that the peer review process CANNOT WORK. DOES NOT WORK, and QUANTIFIABLY, given the record, DOES NOT WORK. Furthermore, he or she would understand that only FALSIFICATION, not correspondence with a model, is the means of proving a theory.

    Since these two problems DISCOVERY (Physical Science) and INVENTION (induction) are the two fundamental problems of the universe, it is not surprising that we are still incompetent at both.

    What is surprising is that in both PHYSICAL SCIENCE and in ECONOMIC SCIENCE, the cognitive bias we bring to our studies, in which we confuse the practical utility of the limited tools and methods of our craft, consistently overwhelms and suppresses the knowledge that in fact, out tools are rough approximations with very poor records of prediction. And that only a combination of prediction and falsification demonstrate the veracity of any theory in either domain.

    A financier who violates one of these principles, or a lawyer, or a craftsman, is held accountable for violating the ethics of his craft. With free speech, comes the same ethical constraint on Physical and Economic scientists. That is because there is HARM put upon populations whenever our work products are put into the public domain such that they may be used for the purpose of policy. The reason is, that all public policy is the application of VIOLENCE: the forcible taking of resources and the coercion of individual behavior under the treat of violence. The scientist or the economists is appealing for the application of violence to his purposes.

    Therefore, a scientist is operating ethically by publishing FACTS. He or she is NOT operating ethically when he publishes theories or predictions unless the theory is accompanied by falsification. Failure to include how to falsify one’s theories is by definition a form of deception.

    If we made it possible to sue scientists the way we can sue doctors and CEO’s then no doubt the quality of work would increase dramatically. And the fact that we cannot sue scientists for the harm that they cause, puts them in the realm with fortune tellers and astrologists.

    • Stop funding them with the public money!

      • The root of the problem is money – ours.

        This is an international problem, far greater than that Eisenhower anticipated in his 1961 farewell address.

        The money is collected worldwide and distributed by an as-yet-unrecognized international alliance of the world’s political leaders with leaders of the world’s scientific organizations.

        As noted above, “science beyond individual understanding” is an invitation to surrender your mind and soul to this nameless “Big Brother.”

      • “The root of the problem is money – ours.”

        The root of the problem is wrong funding with our taxed money. Use of our money to hide the decline, the raw data, the modeling, the coverups, the whitewash of investigation, to stay on the gravy train for the peudocientists and the universities to get the easy funds.

    • I suspect Hayek would not agree that the answer to faulty scientific forecasts would be to sue the scientists.

      • Gary

        OK. Suing scientists for knowingly and purposely publishing bogus data may be extreme, even if they were funded by the taxpayers.

        But Curt Doolittle’s point is really that there is no “accountability” for a scientist (or team of scientists) who put together something like the hockey stick.

        The worst that can happen is that the paper is thoroughly discredited and the authors have a bit of “egg on their faces” for a short while. But, as we see in the hockey stick case, the individuals are still at work (with taxpayer grants), in some cases still even trying to justify the the discredited work.

        The problem here is that the message conveyed by the bogus study (“unprecedented 20th century warmth for 1300 years”) was just too perfect for IPCC to give up, even once the supporting study had been thoroughly discredited.

        So the message remained and IPCC simply hid the hockey stick deep inside its latest 1,000+ page report rather than giving it “center fold” exposure in the “Summary for Policymakers” report, as had been done prior to its falsification.

        But Curt’s question of “accountability” still remains unanswered.

        Max

      • Gary

        Sorry about “all bold”. (I must have hit the wrong button.)

        Max

    • Curt Doolittle,

      Excellent comment.

    • Curt Doolittle

      Excellent summary!

      Max

    • This is an excellent post!
      Thank you,

  40. This is splendid Dr. C. Every once in a while you put something up that just about brings tears to my eyes. I admit that’s ridiculous, but this is precisely what I’ve been thinking for a long while now. The information is out there. The dissenting papers are out there. The superb long range weather forecasters are out there … with their highly specialized knowledge of how natural climate drivers utterly overwhelm a trace gas essential to life.

    But it takes a fair, and open minded, and above all scientifically curious person to go out and gather all these disparate points of knowledge. Those with a vested interest are certainly not going to do it. Your average progressive type isn’t going to do it either. Why should he? The New York Times is telling him that the science is settled and why would they lie?

    The thing is, they’re not lying. They’re just not aware of all the information that’s out there…

  41. THE SUITE OF ESSAYS FOR SCIENTISTS:

    The Use Of Knowledge In Society – FA Hayek
    http://www.capitalismv3.com/?page_id=1605

    The Pretense Of Knowledge – FA Hayek
    http://www.capitalismv3.com/?page_id=1607

    Sources Of Knowledge And Ignorance – Karl Popper
    http://www.capitalismv3.com/?page_id=1624

    Economics and Knowledge – FA Hayek
    http://www.capitalismv3.com/?page_id=1603

    At least in economics we know our models are false.

    @GaryM : Gary’s bit above is quite accurate and well written.
    What he doesn’t say is that this conflict is taking place within a context of a social movement that is an attempt at seizure of power and prestige by one social class from another, and wherein the USA is an empire, and europe failed and collapsed empires. In an effort to combat mysticism we have raised science and to some degree economics to new forms of religion that are unwarranted by the historical evidence. We have placed extraordinary controls on mysticism and history for political convenience but failed to hold scientists and economists to a similar standard which would prevent them from this ethical abuse of the population.

    Facts. Facts. Facts. Science is the process of producing facts. It is Yeoman’s labor. Yet every scientist wishes to be remembered for a theory. Its is a conflict of incentives.

  42. RE: “Glad to see that everyone is beginning to understand why trust has no place in science.”

    Yes. Trust is an unscientific term.

    Science is calculation. There is no trust involved in calculation. Otherwise it’s not calculation.

    • Wait,

      ‘Trust is an unscientific term.’

      So do you ‘trust’ that you are actually communicating with another person somewhere else in the world when posting that comment? Do you ‘trust’ that your wife/mother is at the other end of a cell phone call? Do you ‘trust’ that sunscreen filled Titanium nano-particles will do more good than bad in the long run?

      So much of our lives is dependent on scientific discovery and truths that I am finding increasingly ironic that so many people are totally willing to ‘trust’ an iPad, but not a climate model. To me, the difference has nothing to do with science. The difference has to do with politics.

      If you have differences of opinion concerning what to do about a scientifically informed issue, that’s one thing. But to make over-generalizations concerning the role of ‘trust’ in science has nothing to do with that.

      It’s also fairly uninformed. The entire scientific process is built on ‘trust’. So such a disposition is flat out incorrect.

      • maxwell,

        “The entire scientific process is built on ‘trust'”

        Thats most deviated comment from science, its faith.

      • Science=Measurement
        Measurement=Calculation
        There is no trust in science.
        if there is, then it’s not science.
        I do not have to trust someone else’s data. I have to question it.

        TRUST is to RATIONALISM as FAITH is to RELIGION
        DATA->FACTS->THEORIES->FALSIFICATION=EMPIRICISM
        Science is empirical. If it’s rational, it’s not science, its philosophy.
        That’s the whole ‘french versus english thing’. The cartesian idiocy versus the anglo empirical.

        Scientific works are only empirical if their falsifiable.

      • Climate models are empirical and jump to the conclusion by advocates of the modeling results which can be twisted to whatever results to taxes on carbon.

      • Here is part of the problem with using Popper/Kuhn/whomever else to define what is and what is not science.

        They are not interested in informing the practitioner on how best to do science. They are not interested in helping society determining what are the best methodologies for producing knowledge that will better people’s lives. They are only interested in some academic activity in which they can argue amongst themselves about what constitutes the history of ‘science’.

        Therefore, any use of their philosophy in the context of actual science is ultimately flawed because that was never their intention.

        Moreover, in a practical setting, ‘trust’ in an important aspect of science, when applied appropriately.

        Case in point. I go to a conference. I hear some talks and view some posters. The research ranges from interesting to ‘how get that get accepted’. I have to make judgments on what seems reasonable because I am not going back to my hotel room to try to reproduce some of the more interesting findings. Because I have to use my best judgment in deciding what seems like worth while avenues to pursue as research in the future (as a reviewer of both papers and grants), there is a level of ‘trust’ necessary in my judgment.

        Off the top, I ‘trust’ that no one has lied to me about whether they have actually done the worked they have claimed to have done. I am not going into a talk or reading a paper for the first questioning someone’s intentions. Second, if they make a controversial claim, I ‘trust’ that they have done everything in their control to account for every other explanation. Third, I ‘trust’ that they will be open to criticism and will try to find the best answers to any problems either I or other people raise in the context of their research.

        Popper/Kuhn/whomever else do not account for much of the social aspects of science, in which ‘trust’ is necessary to keep the gear greased and the whole process moving. Again, moreover, the level of objectivity involved is often highly varying because most research is subject to the judgment of researchers who have incomplete knowledge.

        So to make some claims about an idealized situation in which truth flows directly from highly controlled measurements repeated over and over and over again is so far afield from the actual practice of much of science that I completely ignore them. A person making such claims obviously does not have a firm grasp on how science is done.

        There is a great quote we have on the department t-shirts:
        If we knew what we were doing, it wouldn’t be called ‘research’, would it?

        Nope.

      • RE: “They are not interested in informing the practitioner on how best to do science”

        That is false. They are interested in giving the practitioner the tools of how best to do science:
        0) Identify a problem.
        1) establish facts
        2) create theories
        3) falsify theories
        4) repeat forever, because we will never know if our theories are ‘true’, per se, only which of them is already false. Science paints a negative space, and uses what remains.

        If you want to be a researcher, then perform the yeoman’s labor of simply establishing facts, deducing theories, falsifying them, and repeating. Read Einstein. Look at how he did his work. If instead of a scientist, you are a clerk who runs tests, produces data, and contributes to a body of work over which you have no synthetic comprehension, then acknowledge the fact that you are a modern bricklayer and nothing more. There is no dishonor in being a bricklayer.

        But we don’t ask fundamental questions of bricklayers. We ask them to lay bricks.

        Study a little philosophy. This is one of the most important problems in human intellectual history. It may in fact be the only problem in human intellectual history. Philosophy is a catalog of what does not work by some of the greatest minds in history. If would be folly to think you are wiser than they, without first understanding their arguments.

        Personally, I think that scientists should have to be insured in order to publish work, and be open to lawsuits, and damages like any other manufacturer. It’s an amazingly effective methodology.

      • As if we don’t have a hard enough time convincing people to do science, you want them to get insurance in case their wrong. On top of that, you want to the courts to decide if they’re at fault for being wrong?

        I thought you a libertarian, not someone who’s willing to abuse the government’s power to impose your personal preferences on society.

        More to the point, again, you’re conflating the philosophy of something with its practical application. Not surprising of ideologue, but troubling nonetheless.

        I really liked the bricklayer comment. Especially coming from someone who has done next to nothing to falsify he predispositions in this conversation.

      • The 1st ‘their’ = ‘they’re’. Sorry for any confusion.

        It can be taxing taking time to comment between actually doing science.

      • Does one dare venture the ‘glasnost’ middle ground?

        Trust, but verify.

        I’d never trust a vertical supply chain built on trust alone, but one where every node in the chain verified their predecessor and that their predecessor applied this principle, I would expect to be more robust.

        I’d also tend to trust a resilient network model (say an Internet) designed to be tolerant of multiple failures more than a simple bucket-brigade style exchange.

        Science is too large for simple trust, or simple ad hoc verification, but it’s been too large for a long time, and has developed network methods to protect signal integrity.

        It doesn’t always work, as it’s sometimes in the network’s interest to ignore for example hundreds or thousands of weather stations not up to standards, and then it has to have experts painstakingly try to salvage what they can out of the resulting bad data by advanced statistical means and forensic audit methods.. but then, maybe that’s an example of the mechanisms evolved to verify trust?

      • This makes sense to me.

      • Maxwell’s comments, that is. For what it’s worth, when I was reading the “climategate” emails, I saw a lot of overworked scientists.

      • Maxwell, first of all, Popper/Kuhn/whomever else are doing basic research, which by its nature is not intended to have practical application. Second, many people who do applied research on the conduct of science in fact use the results of this basic research. The science of science is no different from any other science in this regard. Basic and applied research are very different and are done by different people.

        For example, I use a Kuhnian model of science to do applied research on the diffusion of scientific thought, in oder to develop better search engines and portals, for scientists to use to improve their work.

  43. The idea that breaking down scientific problems into component parts works just fine for lab science. The good lab scientist attempts to control all variables except the one of interest. But of course, the Earth itself is our lab and many variables are beyond control.

    OK so far. Now take any three environmental variables (predictors) and a response variable of interest (say, crop growth). Now do a multiple regression. Take a fourth explanatory variable, and substitute it for one of your original three environmental predictors. You will find (1) that the values of the coefficients (slopes) has changed for those original variables that were left in the analysis, adn (2) that the R squared has changed.

    You have just demonstrated, statistically, the old cliche that everything is connected to everything else. Unfortunately, we are forced to knwo the environment and ecology primarily throught the use of statistics and correlations. If it was possible to isolate, say, the role of clouds in IR absorption by GHGs, we could come up with a definitive statement on their true role. But of course we can not separate their unique effects from that of a morass of other feedbacks. Scientists love to simplify stuff, but soem stuff just refuses to be simplified.

  44. The text of Michael Nielsen combines several very different examples of scientific complexity in a way that appears to imply similarity. The examples have, however, very little in common and most of them do not seem to fit Nielsen’s message.

    Hayek was discussing how the knowledge of very many individuals and the small decisions they make may lead to solving automatically problems that cannot be solved as well by central planners. That is a positive message to the extent it’s true (and how far it is, remains a major controversy).

    The Large Hadron Collider is a very complex device, and the experiments performed using it are also complex, but this complexity is engineering complexity, not scientific complexity. The theories of elementary particle physics are scientifically complex, but not to the extent that would exceed the mental capacity of its best experts. (Solving the central problems of elementary particle physics may still exceed their capacity, but not mastering the present state of the theories.)

    Neither of the above examples appears to represent that kind of complexity that Nielsen has as his message. The central planners problem for controlling economics might be seen as an extreme example of that, but few economists appear to even propose that they would have a good model for planning real economy (although some statements by some Chicago economists might have been interpreted that way).

    The examples of most complex proofs of theorems of mathematics are perhaps closer to the main message, and the large models of the Earth system (AOGCM’s) fit the picture well. The AOGCM’s may well have reached the point, where nobody understands well enough, how the discretization, local parametrization of clouds, chaotic properties, etc. influence their workings.

    Another point, where the title “Science beyond individual understanding” applies perhaps even more, concerns the relation of the climate science with decision making. How should uncertainties be taken into account, and how all that relates to ethical considerations of sustainable development and intergenerational justice. These issues go beyond science, but there is also a lot of science that should be used to support the decision making. Here the totality appears to exceed very much the understanding of every single individual.

    • Pekka,

      you make several excellent points. Thanks.

    • Even within the proper bounds of “climate science” (the WG1 material), the scientists could stand to benefit from expertise outside of their immediate realm. Statistics, for example. Or control systems dynamics (to an actual control systems engineer, the feedback models used by climate scientists are incredibly naive). Or heat transport (they tend to get the radiation part right, and fumble the convection/conduction part).

      These things could be remedied by a more multidisciplinary attitude, but when McIntyre and McKitrick, for example, attempted to enter the discussion, the “team” put the shields up, locked on the phasers, and lobbed a few photon torpedoes. This is not how multidisciplinarianism is done.

  45. Craig Loehle

    Let’s take the case of particle physics. This is certainly complex and few understand it in every detail. However, they make precise predictions and can do experiments with high precision. You do not need to understand every detail to see that the experiments worked or not. In climate science, the results for let’s say the tropical hotspot show models with a huge range of values and the “matching” or not with theory depends on how you do your testing (and gives a better match the more the models disagree). The hockey sticks are all over the map (why it looks like spagetti). And so on. You do not need to understand every detail to see that the precision is lacking.

    • I am afraid authors’ effort was waste of time, North Atlantic and Pacific do not oscillate, i.e. there are no regular multi-decadal cycles. AMO, PDO & ENSO are products of relatively simple natural processes.
      http://www.vukcevic.talktalk.net/PDO-ENSO-AMO.htm

      • we are still waiting to hear from you what these simple natural processes are. your graphs are interesting, but minus interpretation and explanation, they aren’t very useful IMO

      • I am writing an article entitled ‘Climate Change by Nature’; it will include all data files, physical processes involved, with large amount of additional information in the appendix. When finished I will email you a draft copy.
        Nothing special there, it is all existing knowledge:
        Analysis of data.
        Synthesis of ideas.
        Finally reasoning of an individual.

      • we all look forward to this!

      • Yes – An explanation would be helpful. I keep looking at Vuk’s graphs and can’t make head nor tail of ’em!

      • Dear Dr. Curry:

        Economics is human made and a function of human behavior which is a complex formula. Earth and climate science, on the other hand, is natural and decipherable. The most complex laws of nature were delivered to us in the past by individuals, and earth climate laws will be delivered to us by one single person as well.

        The main-stream science, which is the basis of the so called consensus, is not based on the laws of nature or observations but based on human imagination and fiction. And this is why this main-stream science is so complex, indigestible, and unacceptable.

  46. @Pekka
    (Are you sure you mean chicago? :)

    • I had in mind some statements on the validity of Stochastic Dynamic General Equilibrium Models based on Real Business Cycle theory. I would say that the most obvious overstatements on the quantitative validity of economic theory have been related to these models and come from Chicago. This is not a more general statement for or against any of the disagreeing groups. I refer only to the way people have expressed themselves.

  47. One of the best things S. McIntyre has done is to ask for a reference to any work that explains CO2-forced global warming, from A-Z. His challenge still stands – there is no such work available. People have cited individual papers, which just shows how this basic request can’t be answered. The IPCC reports are just hand-waving exercises, pointing to papers. Where is the single source that goes through the science in an unbroken chain? And how can we upend the entire modern industrial civilization without first seeing it? It takes an outsider, evidently, to ask this simple question. From the profession, all we get is ‘trust us, we know.’

    • As I said before, the central flaw in the IPCC is it’s “underpants gnomes” approach:

      1) Survey the literature
      2) ???
      3) Policy!

      The amazing thing is that it seems to have never occurred to them that there’s anything wrong with this approach.

  48. “The Road to Serfdom” is what the AGW crowd has us on. The control of how much energy we can have at what cost will indeed enslave us. This has never been about AGW it has always been about control of people.

    • Judith – are you reading this?

      • Joshua,

        I won’t attempt to speak for mkelly, but I will point out that there is a very strong strain of anti-capitalist sentiment in the AGW/environmental crowd. A lot of greens are watermelons. Simply look at Maurice Strong’s influence. Or remember the response of the delegates at Copenhagen when Venezuela’s marxist dictator, Hugo Chavez, went off on a rant condemning capitalism — the crowd went nuts with applause.

        This doesn’t mean that every global warming believer is a marxist. To the extent that liberals (as used in the US) are distinguishable from the socialists farther to the left, I am sure that most believers probably aren’t.

      • stan,

        I won’t attempt to speak for mkelly, but I will point out that there is a very strong strain of anti-capitalist sentiment in the AGW/environmental crowd.

        In general, I would agree. I would imagine that we might differ on exactly how strong we see that strain to be, and to what degree anti-capitalism is a driver rather than an influence (along with other influences such as a desire to preserve the environment, advocate for equitable impact of energy/environmental policies, etc.) – but as a general statement I don’t doubt that statement is accurate.

        To the extent that liberals (as used in the US) are distinguishable from the socialists farther to the left, I am sure that most believers probably aren’t.

        Here, however, I get lost in the rhetoric. I don’t know how you define “socialists,” I don’t know what you mean by “believer,” and I don’t know what level of familiarity you actually have with “liberals.” If I knew more specifically what you mean, I could comment on it.

    • I suspect more specifically exploiting greenness to promote policies that favor nuclear energy and trash trading. Where did Keynes say we needed a market for bads?

      The pseudo science in Al Gore’s book “Our choice” is palpable to a scientifically minded reader. A team of writers working for one person who hasn’t read the primary information is guaranteed not to deliver deeper understanding.

  49. I agree with Jerry and Labmunkey’s exchange, beginning here: Jerry | April 20, 2011 at 10:29 pm . I think the author’s point is more that it’s extremely difficult and time consuming to develop a deep understanding (or possibly impossible). The impossibility would come from not having someone who has the breadth and depth of knowledge in diverse areas required to develop the deep understanding. I don’t disagree with this – it seems plausible.

    This is something of an old notion in research management. Old style research management practice was to try to include a polymath, especially as the project manager. Apparently it’s still talked about in some circles:
    http://books.google.com/books?id=1gGSaYzJ-qgC&pg=PA14&lpg=PA14&dq=%22research+management%22+polymath&source=bl&ots=PB0Ioa1noZ&sig=II4C2XFxtCeVV_5Kl_AZ11-hoc4&hl=en&ei=2c-wTaDRKYe5twes4Nn3Cw&sa=X&oi=book_result&ct=result&resnum=1&ved=0CBkQ6AEwAA#v=onepage&q=%22research%20management%22%20polymath&f=false

    Climate research cries out for polymaths, but I don’t see them being used effectively.

  50. @Maxwell: “As if we don’t have a hard enough time convincing people to do science, you want them to get insurance in case their wrong. On top of that, you want to the courts to decide if they’re at fault for being wrong?”

    a) there is no evidence that we are having a problem convincing people to do science. There is evidence that the number of “hard” degrees has maintained constant since the 1960’s. (Per Menand)

    b) the insurance position is a metaphor that helps illustrate the perverse incentives in scientific research. (I look at the data. You may be a practitioner, but the data is the data, so don’t romanticize your craft.) libertarianism, in the sense that it is an extension of classical liberalism, depends first on Juridical defense . Libertarianism in the sense that it is anarchic (rothbardian) depends upon insurance rather than reputation, since reputation is subject to ‘market failure’ – a term which they dislike – but simply means in this case, that prices are insufficient to convey reputation.

    Further I don’t understand your criticism of my position above. You’re consistently confusing, throughout all your postings, the utilitarian and pragmatic pursuit of your craft, with the requirement that policy, which is the application of violence, be dependent upon truth, rather than error, speculation, or fraud. In fact, the only argument you make is that we should sympathize with the toil of the clerical scientist. There is noting special about science. IT is just work, like any other industry.

    If indeed you want to believe science exists in a knowledge market , then you must also accept that markets must be regulated against incompetence fraud and theft, else the market experiences increasing prices and decreasing productivity – corruption. And that is what publishing papers that do not meet quality tests of falsification are: they are a form of corruption or fraud depending upon the case. They are an attempt at personal gain by fraud.

    There is no difference between putting a bit of steam pipe on sale that is dangerous because it is insufficiently tested for the pressure that it must endure and putting a paper into the market of human political discourse – because just as steam is dangerous, so is policy.

    The evidence is in, and it shows quite clearly that the peer review process of self-policing is ineffective. The evidence is in that we produce more graduates and phd’s than the market can absorb. The evidence is in that their works are far too often political rather than empirical. The evidence is in that the long form of publication is superior to the paper/article process. And it certainly appears that the perverse incentives to publish are the cause.

    For that reason, frankly, I do not understand why scientists get a pass on competence. We do not give the private sector a pass. We do not give soldiers a pass. We don’t give lawyers a pass. We don’t give accountants a pass. We don’t even give the average carpenter a pass. The only people we give a pass to are journalists and scientists, and we do so because we believed that the good outweighed the bad, not because there is no ‘bad’ – and by any measure, the craft of scientists is no more ethically represented from that of journalists. There are ethical scientists. There are ethical journalists. But there are enough that are not to scandalize all.

    Economists (other than Krugman, who is perhaps the most dishonest hack in the business) play this game all the time, because we’re dealing with money and politics and people take it deathly seriously. There is no reason, that in matters of policy, we do not hold scientists to the same standard.

    • Yup, dat dere Climategate thingy done bust a gasket in de whole machine.
      ================

    • Prezakly. Well spoked.

    • Curt, that is a BRILLIANT post.

      Thank you.

      …the utilitarian and pragmatic pursuit of your craft, with the requirement that policy, which is the application of violence, be dependent upon truth, rather than error, speculation, or fraud.

      1) ERROR
      The verification period, the biggest “miss” was an apparently very warm year in the late 19th century that we did not get right at all. This makes criticisms of the “antis” difficult to respond to (they have not yet risen to this level of sophistication, but they are “on the scent”).
      http://bit.ly/ggpyM1

      2) SPECULATION
      Yeah, it wasn’t so much 1998 and all that that I was concerned about, used to dealing with that, but the possibility that we might be going through a longer – 10 year – period [IT IS 13 YEARS NOW!] of relatively stable temperatures beyond what you might expect from La Nina etc. Speculation, but if I see this as a possibility then others might also. Anyway, I’ll maybe cut the last few points off the filtered curve before I give the talk again as that’s trending down as a result of the end effects and the recent cold-ish years.
      http://bit.ly/ajuqdN

      3) FRAUD

      I would not give them *anything*. I would not respond or even acknowledge receipt of their emails. There is no reason to give them any data, in my opinion, and I think we do so at our own peril!

      http://bit.ly/8SPNry

  51. Judith Curry

    Let me give you my (unsolicited) additions to your comments on the Nielsen essay, which “adapts the concept of hidden knowledge to science”:

    In economics, many basic facts, such as prices, have an origin which isn’t completely understood by any single person, no matter how bright or well informed, because none of those people have access to all the hidden knowledge that determines those prices.
    By contrast, until quite recently the complete justification for even the most complex scientific facts could be understood by a single person.
    Science is no longer so simple; many important scientific facts now have justifications that are beyond the comprehension of a single person.

    JC: This is certainly true of the climate problem in all its complexity.

    Yes. This may be so. But, as a field that is still in its infancy, climate science has a much greater complication than “complexity” alone, and that is “uncertainty” (as you have emphasized many times in the past). On top of this comes the even larger problem of “advocacy”, which turns pieces of the “science” into sales pitches for a specific political course of action.

    No single person understands all of this, except in broad detail.
    Instead, there will be a large group of people who collectively claim to understand all the separate pieces that go into the discovery, and how those pieces fit together.

    JC: This would explain the IPCC “consensus” and the large number of people that support it.

    Judith, there may well be the “channel vision” of only being rationally skeptical in one’s own specialty; e.g. a sea level expert might simply accept at face value the climate sensitivity estimates of a climate modeler. However, I believe that the “consensus” is more of an issue of “advocacy” than of specialization or scientific “certainty”. The message is the desired message, therefore it is the generally accepted message.

    Two clarifications are in order. First, when I say that these are examples of scientific facts beyond individual understanding, I’m not saying a single person can’t understand the meaning of the facts. I’m talking about a deeper type of understanding, the understanding that comes from understanding the justification of the facts.

    JC: Yes, this aptly describes the missing arguments and justifications from the IPCC report, which instead substitutes citations of papers and lists of evidence, summarized by a confidence level derived from expert judgment.

    Again, Judith, I believe that the underlying problem with all IPCC reports is that their primary agenda is to “sell” the concept of potentially alarming AGW. As a result, those papers are cited which support the desired message while those that do not are either downplayed or ignored completely. Just one specific example of this is the mass balance of both the Greenland and Antarctic ice sheets. Studies covering 365 day/year satellite readings over a full 10-year period from 1993 to 2003 (Zwally/Johannessen and Wingham/Davis) showed net mass gain of both ice sheets over this period. However, these were ignored or simply not mentioned, while results from spot studies covering only parts of the ice sheets or a small piece of the time period, which showed mass loss, were used to support the conclusion that both ice sheets lost mass over this time period, contributing to sea level rise. Unfortunately for the credibility of IPCC, there are many examples of this, Judith.

    Second, I don’t mean that to understand something you need to have mastered all the rote details.

    JC: The emphasis of a certain element of the climate blogosphere on factoids and information and the dismissal of anyone who makes a misstatement is not useful in the context of understanding.

    Judith, I agree that there is a lot of misinformation out there. Unfortunately, some of it is coming from what was once considered to be the “gold standard” climate group, IPCC. And this is the misinformation that is the most damaging. But I agree that one does not have to be an expert in every special field related to climate science in order to understand the overall concepts (just like the CEO of a company does not need to understand all the “rote details” of the R+D work reported by his R+D director to make the right decision about a new product launch).

    How do we know whether [scientific discoveries are] right or wrong? The traditional process of peer review and the criterion of reproducibility work well when experiments are cheap, and one scientist can explain to another what was done. But they don’t work so well as experiments get more expensive, when no one person fully understands how an experiment was done, and when experiments and their analyses involve reams of data or ideas.

    JC: Because of the complexity of many climate studies, peer review is typically cursory in its ability to uncover methodological or logical problems.

    Judith, in addition to the point you mention regarding complexity, “peer review” in climate science has unfortunately become a corrupted process, as recent revelations have shown. It is therefore (as you say) no guarantee of scientific validity or accuracy. This has less to do with whether the “experiment” itself was “expensive” or “cheap”, but with the fact that AGW has become a multi-billion dollar big business.

    Might we one day find ourselves in a situation like in a free market where systematic misunderstandings can infect our collective conclusions? How can we be sure the results of large-scale collaborations or computing projects are reliable? Are there results from this kind of science that are already widely believed, maybe even influencing public policy, but are, in fact, wrong?

    JC: need I actually say it?

    Well, despite the so-called “consensus”, there are a lot of conflicting conclusions on our planet’s climate, so some of the “science” is “wrong” by definition. In addition, in a “free market”, advertising and PR are used to “market” a product. Nisbet’s study showed how billions are being spent to “sell” the “AGW is a potential serious danger requiring immediate action” story (as well as its opposite). Even governments are getting into the act to “sell” the message via TV fairy tales and other forms of scare mongering. The IPCC reports are a bit more subtle, but the same message is being “sold” there. Can this result in “systematic misunderstandings”, which “can infect our collective conclusions”? (If not, why not?)

    But such collaborations will be no good if we can’t assess the reliability of the results. And it would disastrous if erroneous results were to have a major impact on public policy. We’re in for a turbulent and interesting period as scientists think through what’s needed to arrive at reliable scientific conclusions in the age of big collaborations.

    JC: this seems to be what Steve McIntyre is going after, in terms of the auditing concept and engineering-style explanation, and making explicit the “hidden knowledge”.

    Indeed. And the general public will demand more transparency in climate science, as well as more independent audits of key conclusions reached.

    Just my comments to add to yours.

    Max

  52. Dr. Curry,
    This may be a duplicate post. If so, please feel free to delete.

    Often knowledge is not hidden, but rather forgotten.
    Here are some memory jogs, as well as a small social mania algebra primer:

    “• “…civilization will end within 15 or 30 years unless immediate action is taken against problems facing mankind,” biologist George Wald, Harvard University, April 19, 1970.

    • By 1995, “…somewhere between 75 and 85 percent of all the species of living animals will be extinct.” Sen. Gaylord Nelson, quoting Dr. S. Dillon Ripley, Look magazine, April 1970.

    • Because of increased dust, cloud cover and water vapor “…the planet will cool, the water vapor will fall and freeze, and a new Ice Age will be born,” Newsweek magazine, January 26, 1970.

    • The world will be “…eleven degrees colder in the year 2000. This is about twice what it would take to put us into an ice age,” Kenneth Watt, speaking at Swarthmore University, April 19, 1970.

    • “We are in an environmental crisis which threatens the survival of this nation, and of the world as a suitable place of human habitation,” biologist Barry Commoner, University of Washington, writing in the journal Environment, April 1970.

    • “Man must stop pollution and conserve his resources, not merely to enhance existence but to save the race from the intolerable deteriorations and possible extinction,” The New York Times editorial, April 20, 1970.

    • “By 1985, air pollution will have reduced the amount of sunlight reaching earth by one half…” Life magazine, January 1970.

    • “Population will inevitably and completely outstrip whatever small increases in food supplies we make,” Paul Ehrlich, interview in Mademoiselle magazine, April 1970.

    • “…air pollution…is certainly going to take hundreds of thousands of lives in the next few years alone,” Paul Ehrlich, interview in Mademoiselle magazine, April 1970.

    • Ehrlich also predicted that in 1973, 200,000 Americans would die from air pollution, and that by 1980 the life expectancy of Americans would be 42 years.

    • “It is already too late to avoid mass starvation,” Earth Day organizer Denis Hayes, The Living Wilderness, Spring 1970.

    • “By the year 2000…the entire world, with the exception of Western Europe, North America and Australia, will be in famine,” Peter Gunter, North Texas State University, The Living Wilderness, Spring 1970.”

    Scientists + Agenda + Credulous press + social standing (profit) = Crap science.

    Crap science + profit motive + power of belonging = compelling social movement

    compelling social movement + popularity + crap science = Bad politics

    Bad politics + compelling social movement = bad policy

    bad policy + compelling social movement + profit motive = more money for bad science

    more money for bad science + credulous press = compelling crap stories

    compelling crap stories + compelling social movement + bad policy = bad laws
    bad laws = waste of public money + bad outcomes

    The list of failed enviro disaster predictions is from a WUWT post.
    The little primer is from something I am working on.

    • fair enough, but we know A LOT more now than we did in 1970

      • lol Like what?

        Andrew

      • Yes, we do. But everything we’ve learned has also opened new questions and revealed new unknowns.

        Which then means that while we know more, we also know less.

        Which is also as it should be.

      • Dr. Curry,
        Good point, but is there any reason to think the predictions for today’s tomorrow will be any better than the failed ones for today from yesterday?
        How do you like my basic algebra notes, by the way?

      • Yes, but how much of what we know is true?

      • Do we? Yes, I think we do have more DATA. We have more KNOWLEDGE – that is indisputable. But do we have more WISDOM? Data and knowledge appear to be growing exponentially, while wisdom, if it is growing at all, is more closely a linear function. Or so it seems to me. Is this not the essense of the article you cited, Judith? Did they not use “understanding” in place of my use of “wisdom”?

        In climate science, the available knowledge would seem to suggest that we need act NOW. I would suggest that wisdom dictates we need less haste and more speed, to cite the old adage. And for my money, Steve McI’s request for an “engineering quality” exposition on AGW is extremely apropos here, and would do more to convince the sceptical than the current communications stratagy. If we have “wasted” 20 years because of bad commications, will we waste another two decades because such an exposition is too difficault, or takes too long? Where is the wisdom in that?

    • Good points, hunter.

      The fact that some scientists, and academics, and activists from a vast array of fields, many of whom were outliers, were wrong about their predictions years ago, proves that all predictions from any scientists, academics, and activists now, must, necessarily be wrong.

      Well – except predictions from any scientists, academics, and activists that support conservative or libertarian causes.

      Oh. Wait. That wasn’t accurate.

      Except predictions from any conservative or libertarian scientists, academics, and activists who make predictions that hunter agrees with. There, I think that’s better.

      It’s a well-known fact that all scientists, academics, and activists that hunter agrees with have never made any incorrect predictions.

      Don’t forget, reality has a conservative/libertarian bias.

      • “The fact that some scientists, and academics, and activists from a vast array of fields, many of whom were outliers, were wrong about their predictions years ago, proves that all predictions from any scientists, academics, and activists now, must, necessarily be wrong.”

        Misdirection. The argument is that we don’t have any information that indicates a prediction today will be any better than the predictions of yesterday. JC says we know A LOT more, but won’t say about what.

        Misdirection and not bothering to answer questions.

        Seen it all before.

        Andrew

      • The argument is that we don’t have any information that indicates a prediction today will be any better than the predictions of yesterday.

        Would that apply only for the minority of outlier predictions that went against hunter’s ideology?

        Or, would it also apply for the many, many predictions that went against hunter’s ideology that turned out to be correct?

        Would is also apply to the predictions that were in line with hunter’s ideology that proved wrong?

        hunter has quite an impressively long list there of predictions proved false. How do you think the predictions on that list might stack up against a full accounting of all predictions made by scientists, academics, or activists that proved correct? Do you have any idea if it is 1 to 1? 1 to 10? 1 to 1,000,000?

      • Joshua,

        Why don’t you just tell us and cite specifically how all the relevant climate predictions panned out? Show us what you know.

        Anything would be better than the amout of knowledge you’ve shared with us so far… which is zero, as far as I can tell.

        Andrew

      • Or, would it also apply for the many, many predictions that went against hunter’s ideology that turned out to be correct?

        Why don’t you provide a list at least as long as hunters of predictions that turned out to be correct? Until you can do that, you’re just whining.

        Would is also apply to the predictions that were in line with hunter’s ideology that proved wrong?

        Since predictions are notoriously chancy, I’m sure there might be some of those, but only hunter could tell you. You’re asking the wrong person.

        hunter has quite an impressively long list there of predictions proved false. How do you think the predictions on that list might stack up against a full accounting of all predictions made by scientists, academics, or activists that proved correct? Do you have any idea if it is 1 to 1? 1 to 10? 1 to 1,000,000?

        If you wnat those answers, you might wasnt to learn to do some research. There’s no reason for anyone else to do it for you – unless you’re willing to pay for it.

      • The fact that some scientists, and academics, and activists from a vast array of fields, many of whom were outliers, were wrong about their predictions years ago, proves that all predictions from any scientists, academics, and activists now, must, necessarily be wrong.

        No, but it does prove that scientists, academics, and activists are not infallible – as they claim to be. They have, do and will be as human, fallible and sometimes outright stupid as any other slice of humanity.

        Don’t forget, reality has a conservative/libertarian bias.

        Actually, that’s true. If Mother Nature was a liberal, there’d be no earthquakes/tsunamis because she would make the world safe.

      • When you show me a scientist, academic, or activist claiming infallibility, Jim, I will join you in laughing at their arrogance.

        And then I’ll point out to you the myriad arrogant posts of “deniers/skeptics” that make absolute assertions about the infallibility of their opinions on climate change, and we can enjoy another good laugh together.

        Speaking of which, did you watch that John Coleman video yet? A regular laugh riot of absolutist arrogance, that one.

        Take for example his statement that ““A Scientific American poll shows that science readers at least have abandoned the global warming bandwagon.”

        Brought tears to my eyes.

      • I thought I was wrong once, but it turns out I was mistaken.

      • That’s funny. I’m going to borrow that one.

      • IIRC, that was originated by Einstein.

      • Gosh, Josh – you must have missed the pronouncements on this blog several months ago about how the models were complete and had included ALL the possible factors required to provide certainty in the output. How is it that you missed that memo?

        Now if you want to provide additional hyperventilation fodder like you did with Watts, go for it. I DID laugh – at your naievete. I learned long ago to demand DETAILs whenever liberal/progressives start to hyperventilate cause it’s nearly always over something minor or non-existent.

        As for Coleman and SA – if you examined the numbers that came out of that poll, you’d find that what he said wasn’t just hyperbole. It wasn’t as definitive as some people thought, but it still shook the SA tree because the words he used were apparently true. At least if the SA whine presented real informaton.

  53. I find it interesting that Joshua fails to make any connection between the climate debate/skepticism and his intended point. I will however, give him credit for presenting this article –

    http://www.wired.com/print/techbiz/it/magazine/17-03/wp_quant

    The following are non-random quotes from the article – with comment added (generally just a restatement of the quote). I’m sure others will see other applications.

    “the correlations between financial quantities are notoriously unstable.”

    The correlations between tree rings and temperature are notoriously unstable. As Briffa et al were warned prior to using them to generate the Hockey Stick.

    “The corporate CDO world relied almost exclusively on this copula-based correlation model,”

    The climate science world relied almost exclusively on CO2-based correlation models. And it worked no better for the climate world than it has for the financial world.

    “Everyone was pinning their hopes on house prices continuing to rise,”

    The IPCC was pinning their hopes on temperatures continuing to rise in concert with CO2.

    They didn’t know, or didn’t ask. One reason was that the outputs came from “black box” computer models and were hard to subject to a commonsense smell test.

    I think that needs no comment.

    Another was that the quants, who should have been more aware of the copula’s weaknesses, weren’t the ones making the big asset-allocation decisions. Their managers, who made the actual calls, lacked the math skills to understand what the models were doing or how they worked. They could, however, understand something as simple as a single correlation number. That was the problem.

    Another was that the scientists, who should have been more aware of the model weaknesses, weren’t the ones making the big asset-allocation decisions. The politicians and bureaucras who made the actual calls, lacked the math skills to understand what the models were doing or how they worked. They could, however, understand something as simple as a single correlation number. That was a problem.

    Nassim Nicholas Taleb -. “Co-association between securities is not measurable using correlation,” because past history can never prepare you for that one day when everything goes south. “Anything that relies on correlation is charlatanism.”

    “Co-association between variables is not measurable using correlation,” because past history can never prepare you for that one day when everything goes south. “Anything that relies on correlation is charlatanism.”

    In the world of finance, too many quants see only the numbers before them and forget about the concrete reality the figures are supposed to represent.

    In the world of climate, too many scientists/modelers see only the numbers before them and forget about the concrete reality the figures are supposed to represent.

    They think they can model just a few years’ worth of data and come up with probabilities for things that may happen only once every 10,000 years. Then people invest on the basis of those probabilities, without stopping to wonder whether the numbers make any sense at all.

    They think they can model just a few years’ worth of data and come up with probabilities for things that may happen in 100 (or 1,000) years. Then people believe on the basis of those probabilities, without stopping to wonder whether the numbers make any sense at all.

    • Jim – I’m assuming that you posted this on the wrong thread? And perhaps you should go back to the proper thread and read again, as I said that the problems with economic modeling have implications regarding climate modeling (or any type of computer modeling for that matter).

      I would never discount the criticism that computer modeling is not foolproof. You must have me mistaken for one of those “true-believers” or “warmists” that you and your buds are always prattling on about. And I’ll add that you missed the part of the article where it spoke about the problem of missing negative feedbacks.

      The value of computer modeling must be taken in full context. It is not a panacea, but neither is it worthless – it’s all about placing the models into the correct risk assessment framework that examines questions of uncertainty (in all directions). Simply rejecting computer modeling for the sake that it is computer modeling (which has inherent problems) would be “denial” that in fact, quants made a whole lot of people a whole lot of money before the bubble burst precisely because of an underevaluation of risk. Writ large, the entire article is essentially about the underestimation of risk.

      And you’ll notice that the article suggests significant implications about the risks associated with assumption of stability as well.

      Just because you want to read the implications of the article in one way only, Jim, you shouldn’t assume the same about anyone else.

      • Yup – wrong thread, Josh. But it’ll do.

        When you show me a scientist, academic, or activist claiming infallibility, Jim, I will join you in laughing at their arrogance.

        So start laughing.

        I’ll add that you missed the part of the article where it spoke about the problem of missing negative feedbacks.

        I had enough to keep me busy. Didn’t need to use “everything.”

        the entire article is essentially about the underestimation of risk.

        Really? I got a lot more out of it – like the hubris of expecting models to replace common sense. Among other things. And yes – I do know about models, Josh.

        And you’ll notice that the article suggests significant implications about the risks associated with assumption of stability as well.

        Yup – that also applies to “climate”. And many of the “warmists,” as you call them, assume that the climate should (MUST) be stable because it’s been that way since – oh, at least last week? And therefore we need to fight climate change. Do you believe that, Josh? Or are you smarter than that?

        Just because you want to read the implications of the article in one way only, Jim, you shouldn’t assume the same about anyone else.

        I made no assumptions about how anyone would read it – but there was a high probablity that you’d read it differently.

    • And Jim – you might want to check out that clip from the MIT dude. He includes a nice discussion about the problems resulting from an assumed lack of correlation.

      And you really should watch the very end where he addresses his audience. You are sure to absolutely love it.

    • “The corporate CDO world relied almost exclusively on this copula-based correlation model,”

      The climate science world relied almost exclusively on CO2-based correlation models. And it worked no better for the climate world than it has for the financial world.

      “Everyone was pinning their hopes on house prices continuing to rise,”

      The IPCC was pinning their hopes on temperatures continuing to rise in concert with CO2.

      Here is the data: http://bit.ly/fUwNN2

  54. Science is no longer so simple; many important scientific facts now have justifications that are beyond the comprehension of a single person.

    JC: This is certainly true of the climate problem in all its complexity.

    Scientific FACTS justified beyond the comprehension of ANYONE?
    This is “certainly true”? I think not. Indeed, if their justification is beyond comprehension, they cannot be FACTs. They do not even rate as testable hypotheses.

    On a different tack, if complexity becomes difficult to understand, perhaps the mental model is wrong.

    The Ptolemaic Epicycles became more and more complex with greater data. Copernicus proposed a simpler model, which didn’t fully pass peer review until after Galileo’s death.

    • “Science is no longer so simple; many important scientific facts now have justifications that are beyond the comprehension of a single person.”

      This was always true. If you had all the scientific knowledge of the properties of a mosquito, could you predict which person in a room it would bite next? Is the climate 50 or 100 years in the future somehow less complicated? A surprising number of scientists that should know better believe it is. They still see the world as the clockwork Victorian age view of high school physics, where everything is predictable if you know the starting variables.

      In that case there would be no such thing as free will. Our futures would already be written, and we ourselves would simply be living out lives predetermined by the starting variables. Existence would have no meaning as we would all be slaves to the starting position of the universe.

      Those that are Christians will perhaps recognizes this as a fundamental breach of God’s promise to mankind, that free will does exist, that our fates are not written. Free will means that the future cannot be predicted in any meaningful fashion, because it would mean that none of us are free to change the future by our actions today.

      Regardless of what any computer model may say, chaos tells us the free will does exists. that any one of us tomorrow may initiate some chain of events that completely invalidates all current projections for the future. And there is nothing that limits free will to humans alone. Is the flight of the mosquito predetermined, or does the mosquito choose who it bites next?

      • The statement in question was about “scientific facts”, not predictions. So you did not address the focus of my post.

        According to quantum mechanics, you increase the precision in the location of a Particle you loose precision on its velocity and ability to predict its future location. So Free Will in no danger.

        Yet in your reply you imply that it is “always true” that predictions are too complex to be understood by one person, but they are capable of being understood by a collection of people. The first part of that is obviously false because individuals state predictions all the time, so it is almost never true.

        Might there exist some predictions that are too complex to be understood by any one person? An evolution of a billion cell climate model over time might be one. If it is too complex to be understood by one person, where it the proof it can it be understood by a collective? Even so, what use are such model results until the salient points are distilled down to testable predictions that are understandable by individuals who can make decisions?

      • science has very few facts. what there are is observations, theories and laws.

        the problem is that many scientists have come to believe that their pet theories are facts. scientific theories are not facts. scientific facts are called laws.

        So, if we are discussing “scientific facts” then we are discussing the laws of science, not the theories of climate science. However, from what I read we are discussing the theories of climate science dressed up as facts, asking questions about complexity.

        The complexity comes from mistaking theory for fact.

      • Ferd, I think we are getting closer to agreement.
        The statement in question:
        many important scientific facts now have justifications that are beyond the comprehension of a single person.

        You said: scientific facts are called laws. I agree.

        I state simply that (A) any scientific fact or law MUST be comprehensible by some individuals. If not, it cannot be accepted as a fact, law, nor even a theory. By “some” I mean more than one that are not in collusion.

        This might seem trivial, but if you cannot accept (A), then Girma’s 4/22 Translation below is the consequent. “You are only an individual. You are incapable of understanding the facts discovered by this committee.” A Fact determined by committee where individual dissent is disregarded because it is mentally impossible. In a free society, this must never be allowed to happen.

        Why do I make an issue of this? Our host, JC, agreed Nielsen’s premise is “certainly true.” I hope she reconsiders.

      • I may have given short shrift to “justification” in the Nielsen’s premise, but I cannot separate the ability to comprehend the fact and comprehend the justifications of the fact. Science is more that rote memorization of laws.

        For instance the 4 color map theorem is simple and its proof difficult, but that does not mean the justification cannot be comprehended by an individual.

        It might take a committee to create the justification for a law. It took the combined works of many mathematicians to prove Fermat’s Last Theorem. Once justified, however, it is understandable by individuals.

  55. Science is no longer so simple; many important scientific facts now have justifications that are beyond the comprehension of a single person.

    TRANSLATION:

    Individuals, don’t argue about global warming with government departments.

  56. In this debate, “hidden knowledge” is really just poor science with too many assumptions, too many unknowns, overblown confidence, and not enough vocal challenges to the mainstream groupthink.

  57. Dr Curry,

    “The mechanism for the 7%/K water vapor increase is described in my feedback chapter, see section 13.3 and esp following eq (13.23)
    http://curry.eas.gatech.edu/climate/pdf/Ch13_GalleyC.pdf

    There is a jump from equation to the anwser. How did you get 7.7%/K in eq (13.23)? Thanks for enlightening.

    • Well the only actual variables are Wv (water vapor path) and greek capital gamma (lapse rate) and To (surface temperature), the rest are constants (like specific gas constant of water vapor, gravitational acceleration). So if you plug in climatological values of the water vapor path, average surface temperature, and average lapse rate, you will get a number like 7.7%, depending on exactly which values you put in. This is arithmetic. if you buy the entire book and go through the earlier chapters, all this is pretty straightforward.

      • Thanks for the reply.

        “So if you plug in climatological values of the water vapor path, average surface temperature, and average lapse rate, you will get a number like 7.7%”

        So what did you use for the values of the water vapor path, average surface temperature (288K?) and average lapse rate? TIA.

      • well this is a good example for you. See how difficult/easy it is for you to get the same number that I did by plugging in different numbers for these variables. This is a microcosm of the process of trying to tune a climate model to get 3C sensitivity (which is orders of magnitude more challenging).

      • Hidden knowledge?

        I found a lot of AGWers say something then conclusion without substantiation or not providing data for verifications. I know how frustrated Steve Mcintyre’s situation.

        Doubling CO2 and then suddently there is a conclusion of 3.7W/m2 radiation addition to the sun’s energy, no substantiation.
        Doubling CO2 and then there is a conclusion of 1 K temperature rise, no substantiation. What happened to the climate community science?

      • The substantiation for 3.7 W m-2 is substantial: it comes from well validated radiative transfer models
        http://judithcurry.com/2010/12/05/confidence-in-radiative-transfer-models/

        The conclusion regarding the 1K temperature change is much dodgier, see
        http://judithcurry.com/2010/12/14/co2-no-feedback-sensitivity-part-ii/

      • If 7% evaporation is true with doubling CO2 at a mere 3.7W/m2, then why is 324W/m2 back radiation has no effect on evaporation when direct sun light radiation has effect of 78W/m2 latent heat of water vaporization in K&T’s 1997 Global Annual Mean Radiation Budget. 324W/m2 back radiation should produce 181W/m2 latent heat of water vaporization. It does not make sense.